ÓPTICA

Professor Diminoi
ÓPTICA
Como nós vemos as coisas?
Observação: antes de estudar esse assunto leia todo esse texto, ele vai te ajudar muito a entender Óptica.
“Muitas pessoas têm a impressão de que as coisas estão no seu lugar, olhamos para elas e pronto, já as enxergamos. Mas o processo de ver é muito mais interessante do que isto.”

Primeiro temos que entender sobre a luz, ou melhor, o raio de luz. É uma onda eletromagnética que caminha a uma velocidade absurdamente grande. Ela é da ordem de 3,0.105km/s. Para se ter uma ideia, a luz que vem do Sol leva em torno de oito minutos para atravessar o espaço e chegar na terra. Portanto a imagem do Sol que vemos é de oito minutos atrás. Já é coisa do passado. Louco isto, não é?

Um ponto importante é saber que a luz branca é a fusão das luzes que compões as cores do “arco-íris”. Sim, a luz branca é branca pois tem a mistura de várias “luzes” de cores diferentes. Isto é demonstrado em um experimento chamado “Disco de Newton”, onde um disco com as cores do arco-íris é girado bem rápido e forma a cor branca, neste caso, um branco-gelo. Para observar as cores que formam a luz branca, pegue uma caneta que tem seu corpo transparente.
"Disco de Newton"
As canetas mais antigas e populares são ótimas para esta experiência. Leve a caneta sobre o nariz e próximo ao olho. Olhe na direção de uma lâmpada fluorescente branca. Você verá através do corpo da caneta a formação do arco íris, ou seja, os raios de luz separados.

O arco íris que forma quando chove, tem a mesma ideia. Só que a luz atravessa os pingos de chuva e os raios de luz de cores diferentes se separam e assim percebemos o efeito das cores que formam o arco-íris.

Esta luz branca ao bater em um objeto, de acordo com a pigmentação da pintura do objeto, pode absorver parte das cores ou refletir, ou seja, um objeto que você vê amarelo é porque a luz branca incidiu nele, todas as cores foram absorvidas (azul, verde, vermelho, etc) exceto o amarelo que refletiu.

Este raio de luz amarelo que refletiu é que vai para o seu olho. Por isto você vê a cor amarela. Ele entra pela pupila, atravessa o cristalino e vai até a retina. Calma, vou explicar.

O nosso olho é também chamado de globo ocular, “uma caixa” na forma próxima de uma esfera ou bola.

A pupila é uma janela. Sim, um “buraco” por onde a luz entra, mas com uma proteção na frente como se fosse o “vidro da janela” que é a córnea. A pupila é preta porque está escuro dentro do olho. Se tivesse luz lá dentro, ela ficaria clara, emitindo a luz que sairia lá de dentro. Exatamente como uma janela de uma casa. Já reparou que um médico oftalmologista (de olho) chega bem próximo de você no exame e joga uma luzinha no seu olho. Ele quer ver através da pupila, janela, como está a parte interna do seu globo ocular.

O cristalino é uma lente natural que nós temos em nossos olhos. Quando usamos óculos é porque o cristalino não está numa condição tão boa de focar os objetos e precisa de uma ajudinha.

A retina fica no fundo do olho, onde a luz chega e consequentemente a imagem do que se quer ver. Lá existem várias células chamadas foto sensíveis e que absorvem este raio de luz e geram pulsos elétricos. Estes, através do nervo óptico levam estes pulsos para o nosso cérebro que interpreta estes pulsos como imagens.

Portanto o nosso cérebro não vê as coisas, ele recebe eletricidade, pulsos elétricos, que é interpretada como imagens.

Não esqueça que: a imagem chega em nossa retina de ponta cabeça. O nosso fantástico cérebro é que desinverte a imagem.

Importante:

A cor preta é ausência de luz. Quando um objeto tem a cor preta significa que ele absorve todas as cores de luz que recebe e não reflete nada. Em nosso olho, quando não chega luz, o que o nosso cérebro interpreta é a cor preta. Isto explica também porque a cor preta exposta à luz do Sol esquenta mais que as outras cores.

A cor branca é o oposto da luz preta. Ela reflete todas as cores. Por isto é muito clara. Tem muitos raios misturados, portanto muita energia luminosa.

ÓPTICA
É o ramo da física que estuda os fenômenos relacionados à luz. Devido ao fato do sentido da visão ser o que mais contribui para a aquisição do conhecimento, a óptica é uma ciência bastante antiga, surgindo a partir do momento em que as pessoas começaram a fazer questionamentos sobre o funcionamento da visão e sua relação com os fenômenos ópticos.

Luz branca - É uma mistura de sete cores.
 FONTE DE LUZ
É todo corpo capaz de emitir luz, ou seja, todo corpo visível. Aos corpos que emitem luz própria damos o nome de fonte primária ou corpos luminosos e aos corpos que emitem ou difundem luz de uma fonte primária damos o nome de fonte secundária ou corpos iluminados.

Fonte secundária: Lua e todo superfície polida (refletora).

Fonte primária: Sol, Estrelas vela Lâmpada ferro incandescente etc.

Tipos de fonte de luz - As fontes de luz podem ser: puntiforme ou pontual - fontes cujas dimensões são desprezíveis em comparação com a distância a que são observadas, como por exemplo as estrelas ou extensas - fontes de luz cujas dimensões não podem ser desprezíveis em comparação com a distância a que são observadas.


ÓPTICA GEOMÉTRICA

Raio de luz 
É um conceito teórico usado para facilitar o estudo da luz. Ele indica de onde foi emitido (fonte luminosa) e para onde vai.
Feixe e Pincel de luz
Feixe ou Pincel de Luz

Princípios da Óptica Geométrica:
1º Princípio da Óptica Geométrica ou Propagação Retilínea da Luz  - A luz caminha sempre se propaga em linha reta. 

2º - Princípio da Óptica Geométrica ou Independência de raios de luz - Os raios de luz são independentes, podendo até mesmo se cruzarem, não ocasionando nenhuma mudança em relação à direção dos mesmos.

3º - Princípio da Óptica Geométrica ou Reversibilidade dos Raios de Luz - Os raios de luz são reversíveis, isto é o mesmo caminho de idade é o mesmo caminho de volta.

Por exemplo, se vemos alguém através de um espelho, certamente essa pessoa também nos verá. Assim, os raios de luz sempre são capazes de fazer o caminho na direção inversa. 

Velocidade da luz - Durante muito tempo acreditou-se que a propagação da luz fosse instantânea, ou seja, ela seria imediatamente vista por um observador assim que fosse emitida a partir de uma fonte.

No vácuo, a velocidade de propagação da luz, qualquer que seja a frequência ou cor, é de aproximadamente 3,0 x 105 km/s ou 3,0 x 108 m/s.

Observação: em meios materiais, a velocidade da luz é menor que no vácuo.

Ano-Luz - Utilizado na astronomia como padrão para medir distâncias, o ano-luz é a unidade correspondente à distância que a luz percorre no vácuo durante um ano.

Sendo que a velocidade da luz é igual a 300.000 km/s e que um ano tem 365 dias e 6 horas ou 31.557.600 segundos, temos que a distância percorrida pela luz no vácuo em 1 ano é, aproximadamente, 9.467.280.000.000 km (aproximadamente 10 trilhões de quilômetros).

Observação: a medida ano-luz é utilizado para medir distâncias muito grandes.

Exemplo: A estrela Alfa do Centauro, que é a segunda estrela mais próxima da Terra, está a, aproximadamente, 43 trilhões de quilômetros (43.000.000.000.000 km) ou, simplesmente, 4,3 anos-luz. Isso quer dizer que a luz emitida hoje por essa estrela irá demorar 4,3 anos para chegar à Terra.

Propagação retilínea
O princípio da propagação retilínea afirma que a luz se propaga sempre em linha reta. Graças a esse princípio é possível justificar a formação dos eclipses, do dia e da noite, das sombras etc. 
Eclipse lunar - ocorre quando a Lua está na sombra da Terra, ou seja, a Terra está entre o Sol e a Lua. Isso ocorre porque a luz do Sol se propaga em linha reta e, sendo a Terra um meio opaco, não consegue cruzá-la. 

Eclipse solar - ocorre quando a Lua está entre o Sol e a Terra e é explicado pelo mesmo princípio que justifica o eclipse lunar.
Câmara escura
Uma das aplicações da propagação retilínea da luz é a câmara escura. Essa estrutura consiste em uma caixa de paredes opacas. Em uma dessas paredes há um pequeno orifício.

Ao colocar um objeto AB, de tamanho o, na frente da câmara, os raios de luz que partem de AB e atravessam o orifício o, determinam na parede oposta ao orifício uma figura A’ B’ semelhante ao objeto, porém invertida.

Observe abaixo a imagem da vela que se forma ao fundo da câmara.
O que é Luz? – Conceituar luz, como sendo um agente físico, capaz de sensibilizar nossos órgãos visuais, os olhos. É uma forma de energia radiante, que se propaga por meio de ondas eletromagnéticas.

O que é Óptica Geométrica?
A física é dividida em várias partes dependendo do elemento de estudo, deste modo a Mecânica estuda o Movimento, a Termologia estuda o Calor, e a ÓPTICA estuda a luz.

Óptica Geométrica  é o ramo da Física que estuda um conjunto de fenômenos (chamados de Fenômenos Luminosos) que têm como causa determinante o agente físico Luz.

Para estudarmos a óptica Geométrica, devemos conhecer bem alguns conceitos básicos, como Raio de Luz, Pincel Luminoso e Feixe Luminoso. Todos estes itens da Óptica, veremos a seguir:

CONCEITOS BÁSICO DA ÓPTICA GEOMETRICA
Raio de Luz - É um objeto geométrico (uma linha), que traduz a direção e o sentido de propagação da luz, ou seja, é ele que determina aonde a luz irá “aparecer”.

Pincel Luminoso - É um conjunto de raios originados de um mesmo ponto e que apresentam uma pequena abertura angular uma relação aos outros.

Feixe Luminoso - Se define em um conjunto de raios luminosos, cuja abertura angular entre os raios é relativamente grande, de modo a contemplar uma região ampla do espaço.

FENÔMENOS ÓPTICOS - RESUMO DE FÍSICA 
Tanto para Feixes ou Pinceis Luminosos, possuímos classificações, que variam de acordo com a sua orientação e de acordo com o ponto referencial. A seguir iremos observar como são estas classificações:

Cônico Divergente - Os raios de luz divergem a partir de um único ponto e se espalham pelo ambiente. Como exemplo, temos as lâmpadas acesas ou uma vela também acesa.

Cônico Convergente - Os raios de luz convergem para um único ponto P. A luz parte de várias fontes e iluminam um mesmo ponto. Como exemplo para ilustrar este caso, temos as luzes que iluminam um palco de um show, representam um sistema convergente.

Cilíndrico - Os raios de Luz são todos paralelos entre si. O Sol, que é uma estrela, tem tamanho muito grande em relação à Terra, neste caso os raios luminosos saem do Sol e chegam a Terra com pouca abertura angular.


Fontes de Luz - É todo corpo capaz de emitir luz. A palavra FONTE dá a ideia daquilo que origina ou produz efeito, neste caso aquilo que produz a luz. A luz emitida por este corpo pode, ser própria ou refletida por ele e obedecem a uma classificação:

Fontes Primárias - São aquelas que emitem “Luz própria”, isto é, a energia é proveniente de outro tipo de energia, que é transformada em energia radiante luminosa. Essas fontes de luz, por sua vez, podem ser divididas em incandescentes, luminescentes e bioluminescentes.

Fontes Incandescentes - São fontes que emitem luz em virtude da transformação de sua energia térmica em energia radiante luminosa. Um exemplo que ilustra essa teoria é o Sol, que possui em sua superfície uma temperatura da ordem de 6000ºC, (quente pra caramba).

Ou se você preferir algo mais próximo do nosso dia a dia, temos as Lâmpadas de incandescência (atualmente quase em desuso), cujo filamento de tungstênio, chega a uma temperatura superior a 2000ºC.

Fontes Luminescentes - São fontes que emitem Luz, embora neste caso, as temperaturas são relativamente baixas. Dividimos estas fontes em dois tipos:

Fontes Fluorescentes - Nesta situação, as fontes emitem luz, enquanto houver a atuação de um agente excitador. Um exemplo são as lâmpadas fluorescentes, que comparadas com as Lâmpadas de incandescência possuem temperaturas muito menores.

Fontes Fosforescentes - Neste caso são fontes que emitem luz durante certo tempo mesmo após ter cessado a ação do agente excitador. Exemplos clássicos, deste tipo de fonte são os relógios com mostradores luminosos que permitem a visão no escuro.

As substancias depositadas sobre os algarismos absorvem energia luminosa durante o dia e são capazes de emitir tal energia durante a noite.Fontes Bioluminescentes - Estas fontes tem sua origem nas reações químicas geradas no corpo de alguns animais. Poderemos observar tais reações em algumas formas de inseto, os Vagalumes ou até mesmo animais que vivem nas regiões abissais do fundo do mar.

RESOLVIDOS – SOMBRA E PENUMBRA
01) (UNIRG - TO) O esquema a seguir representa um eclipse solar, no qual a Lua, ao passar entre a Terra e o Sol, produz regiões de umbra (cone de sombra), penumbra e antumbra. Na região da umbra, o eclipse é total (A), na região de penumbra, o eclipse é parcial (C) e na antumbra é anular (B).

Essas regiões acontecem porque os raios que partem do Sol
(A) são independentes.
(B) interferem-se ao passar pela Lua.
(C) são reversíveis.
(D) propagam-se retilineamente.
Resolução:
A sombra, a penumbra, a umbra e a antumbra são fenômenos que só ocorrem por causa do princípio da propagação retilínea da luz.
Alternativa: D

02) (UFF - RJ) Para determinar a que altura H uma fonte de luz pontual está do chão, plano e horizontal, foi realizada a seguinte experiência. Colocou-se um lápis de 0,10 m, perpendicularmente sobre o chão, em duas posições distintas: primeiro em P e depois em Q. A posição P está, exatamente, na vertical que passa pela fonte e, nesta posição, não há formação de sombra do lápis, conforme ilustra esquematicamente a figura.
Na posição Q, a sombra do lápis tem comprimento 49 (quarenta e nove) vezes menor que a distância entre P e Q. A altura H é, aproximadamente, igual a:
(A) 0,49 m
(B) 1,0 m
(C) 1,5 m
(D) 3,0 m
(E) 5,0 m
Resolução:
Ao incidir luz sobre o lápis na posição Q, pode-se perceber a formação de um triângulo retângulo com a sombra que vai de Q até R.

Se a distância QR vale x, a distância PQ é quarenta e nove vezes maior, portanto, vale 49x. Aplicando a ideia de semelhança de triângulos, teremos:

Alternativa: E

03) Um professor de Física perguntou a seus alunos como era possível a ocorrência de eclipses. Ele obteve as seguintes respostas:
João: O eclipse solar ocorre quando a lua posiciona-se na região de sombra da Terra. Para que o eclipse ocorra, os três astros devem estar alinhados.
Lucas: Os fenômenos da sombra e da penumbra explicam a ocorrência dos eclipses.
Aline: O princípio da propagação retilínea da luz possibilita a ocorrência dos eclipses.
Marque a alternativa correta quanto às respostas dadas pelos alunos.
(A) Os três alunos estão corretos.
(B) Aline a João estão errados.
(C) Somente João está errado.
(D) Lucas é o único correto.
(E) Os três alunos estão errados.
Resolução:
Quando a lua posiciona-se na região de sombra da Terra, ocorre o eclipse lunar, e não solar.
Alternativa: C

04) Para as proposições abaixo, marque V para verdadeiro e F para falso.
(   ) O princípio da reversibilidade da luz explica a ocorrência dos eclipses.
(   ) O eclipse solar parcial ocorre na região de penumbra.
(   ) Quando um objeto opaco é posto diante de uma fonte de luz pontual, pode-se perceber a formação de sombra e penumbra.
(   ) Só ocorre penumbra caso a fonte de luz seja extensa, tipo de fonte cujas dimensões não podem ser desprezadas.
(A) VVVV
(B) VFFV
(C) FVFF
(D) FVFV
(E) FFVV
Resolução:
Falso – O princípio da propagação retilínea da luz explica a ocorrência dos eclipses.
Verdadeiro.
Falso – Quando um objeto opaco é posto diante de uma fonte de luz extensa, pode-se perceber a formação de sombra e penumbra.
Verdadeiro.
Alternativa: D

05) (FMTM - MG) O Brasil pôde presenciar, durante a passagem do dia 15 ao 16 de maio, mais um eclipse total da Lua, fato comentado por todos os jornais. Observe a manchete:
Céu limpo realça “show” do eclipse
Em termos astronômicos, o eclipse teve início às 22h05, quando o satélite começou a entrar na zona de penumbra causada pelo bloqueio de parte dos raios do Sol. Nessa fase, o fenômeno não é percebido e praticamente não há diferença no brilho da Lua. O eclipse propriamente dito começou às 23h03, quando a Lua foi obscurecida pela umbra (sombra total) da Terra. Nessa fase – que durou até 2h17 – o satélite adquiriu um tom avermelhado devido ao desvio de parte dos raios de luz na passagem pela atmosfera terrestre.
(O Estado de S. Paulo, 16.05.2003, adaptado)



 No fenômeno observado aproximadamente às 0h12, em uma analogia com uma sala onde a única fonte de luz é a de uma lâmpada presa ao teto, é correto associar o Sol à lâmpada da figura:
(A) 2, a mesa ao planeta Terra e um dos pontos C à Lua.
(B) 2, a mesa ao planeta Terra e o ponto E à Lua.
(C) 2, um dos pontos D ao planeta Terra e a mesa à Lua.
(D) 1, um dos pontos A ao planeta Terra e a mesa à Lua.
(E) 1, a mesa ao planeta Terra e o ponto B à Lua.
Resolução:
O eclipse descrito no texto é o do tipo lunar, e o trecho “…quando a Lua foi obscurecida pela umbra (sombra total) da Terra” evidencia tal ideia. Nesse tipo de eclise, a Lua entra na região de sombra da Terra, onde não pode ser atingida pelos raios de luz provenientes do Sol (tipo de fonte extensa). Por isso, a figura 2 apresenta de forma melhor a situação descrita.
Alternativa: B

06) Marque a alternativa correta a respeito dos fenômenos da sombra, penumbra e eclipses.
(A) A região de eclipse solar parcial é aquela onde metade da Lua pode ser vista, após entrar na região de sombra da Terra.
(B) A sombra só será percebida caso a fonte luminosa seja pontual; a penumbra é percebida com qualquer tipo de fonte luminosa.
(C) A sombra é uma região completamente sem luz; na penumbra, alguns pontos apresentam luz.
(D) A única diferença entre sombra e penumbra é que esta ocorre com qualquer fonte de luz, e aquela ocorre para qualquer tipo de fonte.
(E) Todas as alternativas estão erradas.
Resolução:
Na sombra, não existem pontos com luz, é uma região completamente escura que surge independentemente do tipo de fonte luminosa. A penumbra surge apenas quando a fonte luminosa é extensa e é uma região onde alguns pontos possuem luz.
Alternativa: C

07) Tomando a figura 2 do exercício número 2, imagine um objeto posto no ponto médio dos pontos D e C. Em qual tipo de região, sombra/penumbra, da mesa esse objeto estará?
(A) Entre os pontos D e C, semelhante ao que ocorre entre os pontos D e E, será uma região de sombra.
(B) Entre os pontos D e C, semelhante ao que ocorre entre os pontos D e E, será uma região totalmente iluminada.
(C) Assim como não há sombra entre os pontos C e D, também não existe sombra em D e C.
(D) A região entre os pontos D e C é iluminada pela fonte extensa e também está na região de sombra da mesa, por isso essa região é de penumbra.
(E) Entre os pontos D e C, existe a mesma sombra presente entre os pontos C e D.
Resolução:
O ponto médio entre D e C, assim como entre C e D, ficará em região de penumbra.
Alternativa: D

MEIOS DE PROPAGAÇÃO DA LUZ

A luz pode ser propagada em três diferentes tipos de meios.
Transparentes – Esses meios permitem a passagem ordenada dos raios de luz, dando a possibilidade de ver os corpos com nitidez.
Translúcidos - Nesses meios a luz também se propaga, porém de maneira desordenada, fazendo com que os corpos sejam vistos sem nitidez. Exemplos: vidro fosco, plásticos, etc. 
Opacos – Esse meios são aqueles que impedem completamente a passagem de luz, não permitindo a visão de corpos através dos mesmos. Exemplos: portas de madeira, paredes de cimento, pessoas, etc. 
Observação: quando os raios de luz incidem em uma superfície, eles podem ser refletidos regular ou difusamente, refratados ou absorvidos pelo meio em que incidem.

FENÔMENOS ÓPTICOS

Reflexão - E um fenômeno óptico que ocorre quando a luz incide sobre uma superfície e retorna ao seu meio de origem. Esse processo pode ser classificado como regular ou difuso.

Reflexão Regular – Nesse tipo de reflexão, os raios refletidos ficam paralelos uns aos outros. É esse tipo de reflexão que forma a imagem de superfícies altamente polidas, como os espelhos, metais ou a superfície de um lago. A imagem que se forma nesse tipo de superfície é altamente nítida, porém, ela não pode ser observada de diferentes posições. Pense em um espelho, dependendo da posição que você estiver, não conseguirá ver sua imagem.
Reflexão Difusa - Esse tipo de reflexão ocorre quando a luz incide sobre uma superfície irregular e esta a reflete. Os raios de luz refletidos propagam-se em várias direções diferentes. Os objetos difundem a luz que recebem, por isso, quando essa luz penetra em nossos olhos, conseguimos enxergá-los. Como na difusão a luz propaga-se em todas as direções, muitas pessoas podem ver o mesmo objeto ao mesmo tempo. É por causa desse fenômeno que conseguimos enxergar vários objetos simultaneamente e de ângulos diferentes.
Leis da Reflexão

Reflexão total - É um fenômeno óptico que ocorre quando a luz incidente sobre uma superfície que separa dois meios, no sentido do maior para o menor índice de refração, é totalmente refletida, permanecendo no meio de origem. Esse fenômeno só ocorre se o ângulo de incidência for maior que o chamado ângulo limite.
Ângulo limite - É o menor ângulo de incidência necessário para que o ângulo de refração de um raio de luz, na passagem de um meio com maior para outro com menor índice de refração, seja de 90º.
O índice de refração do meio 2 é maior que o índice de refração do meio 1. Assim, por meio da lei de Snell-Descartes, é possível determinar o ângulo limite L.
O valor do seno do ângulo limite é definido pela razão entre o menor e o maior índice de refração.

Reflexão total da luz -  Caso o ângulo de incidência de um raio de luz seja superior ao ângulo limite, o raio de luz sofre reflexão total.
Observe que o ângulo de incidência do raio de luz verde na imagem acima é maior que o ângulo limite, por isso, o raio de luz foi totalmente refletido, permanecendo no meio 2.

Efeitos da reflexão total

Miragens - Em dias de calor intenso e muito ensolarados, é possível observar a aparente formação de poças d’água no asfalto quente. A imagem vista em pontos distantes no asfalto é uma miragem, resultante do fenômeno da reflexão total.
As várias camadas de ar existentes acima do asfalto possuem diferentes temperaturas. Quanto mais próximo do chão, maior é a temperatura do ar. A densidade das camadas de ar mais quentes é menor e, por isso, o índice de refração das massas de ar vai diminuindo à medida que se aproximam do solo. Portanto, a luz do Sol sofre inúmeras refrações ao se aproximar do chão. Assim, em determinados pontos, o ângulo de incidência dos raios solares torna-se maior que o ângulo limite, provocando a reflexão total da luz. Os raios que resultam da reflexão total atingem os olhos de um possível observador e dão a sensação da formação das poças d’água.

Fibras ópticas 
São filamentos capazes de transportar luz e são utilizadas para diagnósticos de imagens e transmissão de dados.
As fibras ópticas são constituídas por materiais com diferentes índices de refração, o que garante a ocorrência do fenômeno da reflexão total e possibilita a transmissão dos raios de luz.
VELOCIDADE DA LUZ

A velocidade da luz depende do meio em que ela se propaga. Porém adotamos uma constante quando nos referimos a aplicações a diversos outros fenômenos eletromagnéticos como raios-x, raios gama, ondas de rádio e tv, é caracterizada pela letra c, e tem um valor aproximado de 300 mil quilômetros por segundo, ou seja:

c = 3 . 10km/s ou  c = 3. 108 m/s

No entanto, nos meios materiais, a luz se comporta de forma diferente, já que interage com a matéria existente no meio. Em qualquer outro meio que não seja o vácuo,  a velocidade da luz v é menor que c.

Observação: em meios diferentes do vácuo também diminui a velocidade conforme aumenta a frequência. Assim a velocidade da luz vermelha é maior que a velocidade da luz violeta etc.

Sol já desceu no horizonte e não está mais visível a nós, mas por causa da curvatura da Terra, ele ainda continua iluminando a atmosfera, também graças às propriedades de refração da luz.
Observação: No século XVII o matemático e astrônomo holandês Snell descobriu uma lei que possibilita calcular o ângulo de refração como também o índice de refração do meio. Em sua homenagem essa lei ficou conhecida com Lei de Snell.

Leis da Reflexão

DIOPTRO PLANO
É um sistema refrator que apresenta dois meios transparentes homogêneos separados por uma superfície
Um lápis dentro de um copo com água parece estar quebrado em virtude da refração da luz

Você já deve ter visto um objeto dentro de uma piscina e percebido que ele não parece estar no lugar que efetivamente está. Também já deve ter percebido que uma colher dentro de um copo com água parece estar torta. Esses dois fenômenos ocorrem por um mesmo motivo: a refração da luz. E é justamente o que ocorre com um dioptro plano.

O dioptro plano apresenta dois meios transparentes e homogêneos separados por uma superfície plana. Como exemplo, podemos citar a superfície de lagos ou piscinas que separa a água do ar. Se estivermos em um dos dois meios que formam o dioptro plano, a visualização que teremos do objeto no outro meio será afetada pela refração da luz.

A relação entre a posição do objeto e a posição da imagem visualizada pode ser obtida se os índices de refração dos dois meios forem conhecidos. Observe a figura a seguir:

Esquema de formação da imagem em um dioptro plano quando o observador está no meio menos refringente

Na figura acima, o observador está no meio mais refringente, ou seja, no meio em que o índice de refração é maior. Observe que os raios de luz provêm da parte inferior da figura, onde o objeto se localiza, e o índice de refração é n2. O ponto I é o local onde a imagem é formada, e o ponto O corresponde à localização do objeto. Temos também dI, que é a distância da imagem até a superfície, e dO, que é a posição do objeto até a superfície.

Observe também o caso em que o observador localiza-se no meio mais refringente:

Esquema de formação da imagem em um dioptro plano quando o observador está no meio mais refringente Título:
Observador no meio mais refringente
Esquema de formação da imagem em um dioptro plano quando o observador está no meio mais refringente

Observe que, nesse caso, a imagem formada está muito mais longe da superfície do que no caso anterior, em que o observador estava no meio menos refringente. Nas duas situações a imagem formada é virtual.

A equação utilizada para calcular a posição da imagem é válida para as duas situações:

n1 = di
 n2     dO


O valor atribuído a n1 e n2 será sempre relativo à localização do objeto:

n1 sempre será o índice de refração do meio em que o objeto se localiza, ou seja, de onde a luz “vem”.

n2 será sempre o índice de refração do meio para onde a luz vai, que é onde o observador está.

Exemplo
Um dioptro plano constituído por uma superfície plana que separa os dois meios. O exemplo mais simples de um dioptro plano é o par de meios “ar e água”, a partir do qual se estudará a vista do ponto imagem virtual P’ de um objeto real P, por um observador O fora d’água e vice-versa.

Um observador vê o peixe mais próximo da superfície por causa da refração da luz na água
Refração
E um fenômeno óptico que ocorre com a luz quando ela muda de meio de propagação como, por exemplo, ar e água. É importante ficar bem claro que esse acontecimento só ocorre quando o feixe de luz se propaga com velocidade diferente nos dois meios.
   
Absorção
É o fenômeno onde as superfícies absorvem parte ou toda a quantidade de luz que é incidida.

Difração
É a propriedade que as ondas têm de contornar obstáculos ou passar por um orifício quando são parcialmente interrompidas por ele. Ou seja, é a passagem de uma onda lumiosa por uma fenda.
Observação: Essa propriedade dos movimentos ondulatórios foi estudada no ano de 1803, pelo médico, físico e cientista inglês Thomas Young, o qual se tornou muito famoso por ter conseguido obter interferência com a luz. Durante um experimento, Young demonstrou que a luz é um movimento ondulatório e que também sofre difração ao passar por um pequeno orifício. De modo, a provar que a difração também acontecia com a luz, Thomas fez com que feixes de luz passassem por uma pequena e estreita abertura e com um anteparo localizado do outro lado ele viu que não aparecia somente uma linha reta, mas um conjunto de várias faixas com diferentes intensidades. Dessa forma, ele acabou por mostrar que a luz, assim como os outros fenômenos ondulatórios, sofria o fenômeno da difração.

Interferência - É um fenômeno típico dos movimentos ondulatórios, ou seja, pode-se obter a interferência com duas ou mais fontes luminosas ou fontes sonoras como, por exemplo, o alto-falante. Esse acontecimento ocorre de acordo com o princípio da superposição de ondas, e pode ser classificado em interferência construtiva e interferência destrutiva.

Interferência construtiva - ocorre quando as ondas tem a mesma fase e possui caráter de reforço, ou seja, há a formação de uma onda maior que as que deram origem.
Interferência destrutiva – ocorre quando as ondas não tem a mesma fase e possui caráter de aniquilação.
No ramo das telecomunicações, o estudo da interferência é muito importante, pois esse fenômeno é um dos fatores responsáveis pela limitação no tráfego das informações, produzindo ruídos e outros tipos de interferências que podem ser reduzidos com certos tipos de modulação. Esse fenômeno também ocorre nas bolhas de sabão: o feixe luminoso ao incidir na bolha sofre interferência tanto na superfície superior quanto na inferior. Em virtude disso, surgem regiões escuras que são as zonas de interferência destrutiva e as regiões claras que correspondem às zonas de interferência construtiva.

Polarização - A luz natural que antes se propagava em todos os planos, agora passa a se propagar em um único plano. 
Polarização da luz solar por reflexão em um lago

O que é uma luz polarizada? É um conjunto de ondas eletromagnéticas que se propagam em apenas uma direção. 

O que é uma luz não polarizada? É um conjunto de ondas eletromagnéticas que se propagam em todas as direções. 
Os polarizadores funcionam como uma fenda permitindo que a luz passe somente em um plano. Se acontecer de dois polarizadores estarem alinhados na mesma direção, a luz passa pelo primeiro, mas no segundo não se vê nada, pois não haverá emergência de luz. O acontecimento da polarização da luz dá evidências claras de que ela é formada por ondas transversais.
Dessa maneira, esse fenômeno só pode acontecer com esse tipo de onda, assim podemos concluir que com as ondas sonoras não acontece polarização, pois as mesmas são do tipo longitudinal.
CORPO NEGRO
É um objeto hipotético que absorve toda a radiação eletromagnética que nele incide: nenhuma luz o atravessa e nem é refletida. Um corpo com essa propriedade, em princípio, não poderia ser visto, daí o nome corpo negro. Apesar do nome, corpos negros emitem radiação, o que permite determinar sua temperatura.

Observação: a cor com maior comprimento de onda é o vermelho, e as cores seguem como no arco-íris, até o violeta, que tem o menor comprimento de onda do espectro visível.

Exemplo - 1
Exemplo - 2

ÓPTICA GEOMÉTRICA - QUESTÕES RESOLVIDAS
01) (FEI) Uma câmara escura de orifício fornece a imagem de um prédio, o qual se apresenta com altura de 5 cm. Aumentando-se de 100m a distância do prédio à câmara, a imagem reduz-se para 4 cm de altura. Qual é a distância entre o prédio e a câmara, na primeira posição?
(A) 100 m
(B) 200 m
(C) 300 m
(D) 400 m
(E) 500 m
Resolução:
i = 5 cm
O = 4 cm
DI = x + 100
Do = 100
Utilizamos a equação:
DI = i
Do  O
x + 100 = 5
x          4
4 (x + 100) = 5x
4x + 400 = 5x
5x – 4x = 400
x = 400
Alternativa: D

02) Um pesquisador precisava medir a altura de um prédio de vinte andares, porém ele não possuía o instrumento de medida necessário para realizar essa medição. Conhecendo o princípio da propagação retilínea da luz, ele utilizou uma haste de madeira de 1 m de altura e, em seguida, mediu a sombra projetada pela haste, que foi de 20 cm, e a sombra projetada pelo prédio, que foi de 12 m.
Calcule a altura do prédio de acordo com esses dados encontrados pelo pesquisador.
Resolução:
o = ?
i = 12 m
p = 1 m
p' = 20 cm = 0,2 m
Aplicando a expressão:
_i_ = p'

o      p
Substituindo os dados, temos:
12 = 0,2

o      1
 o = 12
0,2
o = 60 m
A altura do prédio é 60 m.

03) Um objeto de 8,0 m de altura é colocado na frente de uma câmara escura de orifício a uma distância de 3,0 m. Sabendo que a câmara possui 25 cm de profundidade, calcule o tamanho da imagem formada.
Resolução:
o = 8,0 m
p = 3,0 m
p' = 25 cm = 0,25 m
i = ?
Aplicando a expressão:
_i_ = p'

o     p
Substituindo os dados, temos:
_i_ = 0,25

 8        3
3 i = 8 . 0,25
i = 2
     3
i = 0,67 m

04) (Unitau) Dois raios de luz, que se propagam em um meio homogêneo e transparente, interceptam-se em certo ponto. A partir desse ponto, pode-se afirmar que:
(A) os raios luminosos cancelam-se.
(B) mudam a direção de propagação.
(C) continuam propagando-se na mesma direção e sentindo que antes.
(D) propagam-se em trajetórias curvas.
(E) retornam em sentidos opostos.
Resolução:
Quando dois raios de luz propagam-se em um meio homogêneo e transparente, eles continuam propagando-se na mesma direção e sentido. Isso ocorre graças ao princípio da independência dos raios luminosos, que afirma que os raios de luz são independentes ao interceptarem-se, pois cada um deles mantém sua trajetória como se os demais não existissem.
Alternativa: C

05) Entre as alternativas a seguir, escolha aquela que contém apenas fontes primárias de luz.
(A) Fósforo, Sol, Lua
(B) Lua, Júpiter, Sol
(C) Vela acesa, Sol, Lua
(D) Estrelas, Fósforo aceso, Sol
(E) Estrelas, pilha de lanterna e Sol.
Resolução:
As fontes primárias de luz são aqueles que produzem luz própria. Algumas fontes, como fósforos e lâmpadas, são fontes primárias desde que estejam acesas.
Alternativa: D

06) (UFAL) A figura representa um feixe de raios paralelos incidentes em uma superfície S e os correspondentes raios emergentes:
Essa figura ilustra o fenômeno óptico da:
(A) dispersão.
(B) reflexão difusa.
(C) refração.
(D) difração.
(E) reflexão regular.
Resolução:
A reflexão é o fenômeno em que os raios de luz atingem determinada superfície e têm a sua direção de propagação alterada. Quando os raios refletidos (emergentes) mantêm-se paralelos, a reflexão é chamada de regular; porém, caso a superfície seja rugosa, os raios refletidos não serão paralelos, e a reflexão será denominada de difusa.
Alternativa: B

07) Indique a alternativa que explica de forma correta a diferença entre as fontes de luz fluorecentes e fosforescentes.
(A) As fluorecentes emitem luz a partir da excitação do flúor em seu interior, e as fosforescentes funcionam pela excitação do fósforo.
(B) As fluorecentes emitem luz durante a ação de um agente excitador, e as fosforescentes emitem radiações ultravioleta.
(C) As fluorecentes emitem luz durante a ação de um agente excitador, e as fosforescente emitem luz mesmo após o fim da ação do excitador.
(D) Lâmpadas fluorecentes funcionam a partir da excitação de gases como o argônio, e materiais fosforescentes funcionam por meio da excitação do fósforo.
(E) Os termos fluorecentes e fosforescentes são sinônimos.
Resolução:
As fontes de luz do tipo fluorecente funcionam enquanto houver ação do elemento excitador, que, geralmente, é o argônio. Fontes fosforescentes emitem luz mesmo após o fim da ação do excitador. Seu funcionamento baseia-se na energia potencial química.
Alternativa: C

08) As afirmações a seguir tratam dos conceitos básicos de Óptica Geométrica. Indique a questão incorreta.
(A) Raios de luz são setas orientadas que representam a luz e são classificados como paralelos, convergentes e divergentes.
(B) A Óptica Geométrica estuda a natureza física da luz.
(C) Fontes secundárias de luz são aquelas que não produzem luz própria. A Lua é um exemplo de fonte secundária.
(D) Quando um feixe luminoso muda de meio de propagação, ocorre o fenômeno óptico da refração.
(E) A Óptica Geométrica estuda os fenômenos com base em experimentos e não analisa a natureza física da luz, mas a interpreta como setas orientadas denominadas de raios de luz.
Resolução:
A Óptica Geométrica não leva em consideração a natureza física da luz, mas a interpreta como raios luminosos, isto é, setas orientadas que representam a luz.
Alternativa: B

09) Uma fonte secundária de luz que se apresenta na cor azul possui tal cor porque:
(A) refrata a luz incidente.
(B) reflete a luz azul.
(C) difrata a luz azul.
(D) absorve a luz azul.
(E) emite luz azul.
Resolução:
Fontes secundárias refletem a luz incidente. Se o objeto é percebido na cor azul, é porque ele possui a capacidade de refletir a luz dessa cor.
Alternativa: B

CASO 1: RESOLVIDAS
01) (PUC-RIO) Um feixe de luz de comprimento de onda de 600 nm se propaga no vácuo até atingir a superfície de uma placa de vidro. Sabendo-se que o índice de refração do vidro é n = 1,5 e que a velocidade de propagação da luz no vácuo é de 3 x 10m/s, o comprimento de onda e a velocidade de propagação da onda no vidro em nm e m/s, respectivamente, são: (Obs: 1 nm = 1 x 10−9 m).
(A) 200 nm; 4 x 108 m/s
(B) 200 nm; 3 x 108 m/s
(C) 200 nm; 2 x 108 m/s
(C) 400 nm; 1 x 108 m/s
(E) 400 nm; 2 x 108 m/s
Resolução:
Quando a luz passa de um meio para outro com índice de refração diferente e com o seu comprimento de onda alterado, utilizamos a relação a seguir para calcular o novo comprimento de onda:
λ' = λ0
          
n

Sendo que:
λ' é o comprimento de onda ao passar para um meio material;
λ0 é o comprimento de onda no vácuo;
n é o índice de refração do meio.
Substituindo os dados do problema na equação acima, temos que:
λ' = 600 x 10-9 = 400 nm
        1,5           

E a velocidade da onda é dada por:
v = 3 x 10= 2 x 108 m/s
       n      1,5                      

Alternativa: E

02) (UN. MACKENZIE) A velocidade de propagação da luz em determinado líquido é 80% daquela verificada no vácuo. O índice de refração desse líquido é:
(A)1,50
(B)1,25
(C)1,00
(D) 0,80
(E) 0,20
Resolução:
Inicialmente, é necessário separar os dados oferecidos pelo problema:
c – velocidade da luz no vácuo;
80% c = 0,8c é a velocidade de propagação da luz no líquido.
Utilizando a equação:
n = c
      v
Substituindo os dados:
n =    c  
      0,8c

Cancelando c, temos:
n = = 1,25
     0,8

O índice de refração é 1,25
Alternativa: B

03) Um raio de luz atravessa a interface entre o ar e um líquido desconhecido, mudando sua direção conforme mostra a figura abaixo. Sabendo que o índice de refração do ar é 1, calcule o índice de refração do líquido. Dados: sen35º = 0,57 e sen20º = 0,34.
O raio de luz atravessa a interface entre dois meios e sofre refração
Resolução:
Para encontrar o índice de refração do líquido, devemos utilizar a Lei de Snell:
nar . Senθ = nliquido . Senθ2
Substituindo os dados, temos:
1 . sen35 = nliquido . sen20
1 . 0,57 = nliquido . 0,34
nliquido 0,57
             0,34

nliquido = 1,67

04) A luz atravessa um material feito de plástico com velocidade v = 1,5 x 108 m/s. Sabendo que a velocidade da luz no vácuo é 3,0 x 108 m/s, calcule o índice de refração do plástico.
Resolução:
O índice de refração é calculado com a expressão:
n = c
      v

n =    3 x 108
        
1,5 x 108

n = 2

05) (UERN) Um feixe de luz proveniente de um meio A propaga-se em direção à superfície de separação com um meio B. Se o índice de refração do meio B em relação ao meio A é igual a 1,25, ao sofrer a refração, o feixe de luz teve sua velocidade:
(A) reduzida em 25%.
(B) reduzida em 20%.
(C) aumentada em 20%.
(D) aumentada em 25%.
Resolução:
A relação entre o índice de refração B e A é maior que 1, o que indica que o índice de refração do meio B é maior que o do meio A. Portanto, ao passar para o meio B, a velocidade da luz deverá ser reduzida.

A velocidade da luz no meio B corresponde a 80% da velocidade da luz no meio A, logo, houve uma redução de 20% no valor da velocidade da luz.
Alternativa: B

06) (PUC-Camp-SP) Há uma série de fenômenos observáveis que ocorrem na atmosfera terrestre e que são determinados pela refração da luz ao percorrê-la. São exemplos o arco-íris, a formação de halos ao redor da Lua, a posição aparente dos astros mais elevada que a real, o fenômeno das miragens no deserto, a aurora boreal. A refração é o fenômeno da mudança na velocidade da propagação da luz ao passar de um meio transparente para outro. Considere um raio luminoso que forma ângulos iguais a 45º e 60º com a superfície que separa o vácuo do meio transparente e homogêneo M, como representado na figura.
Dados:
Velocidade da luz no vácuo = 3,0 . 108 m/s
De acordo com a situação apresentada, a velocidade da luz no meio M, em m/s, vale:
(A) 1,0 . 108
(B) 1,0√2. 108
(C) 1,5 . 108
(D) 1,0 .√3 . 108
(E) 1,5 . √2. 108
Resolução:
Por meio da definição de índice de refração da Lei de Snell-Descartes e sabendo que o índice de refração do vácuo é igual a 1, pode-se definir uma relação entre os senos dos ângulos e as velocidades da luz nos meios em questão.

Os ângulos de incidência e reflexão são estabelecidos entre os raios de luz e a reta normal, portanto, na figura do exercício, os ângulos complementares de 45° e 60° representam os ângulos de incidência e reflexão. Sabendo que o ângulo complementar é aquele que completa a soma com resultado igual a 90°, que 45° é complementar do próprio 45° e que 30° é complementar de 60°, os ângulos de incidência (θi) e refração (θR) são, respectivamente, 45° e 30°.

Alternativa: E

07) Um raio de luz monocromático incide na superfície de separação entre dois meios com índice de refração igual a 1,5 e 2,0, passando do menor para o maior índice. Determine o ângulo de refração sabendo que o ângulo de incidência é de 30°.
(A) 14°
(B) 18°
(C) 22°
(D) 26°
(E) 28°
Resolução:
O ângulo de refração será determinado a partir da aplicação da Lei de Snell-Descartes.

Alternativa: C

08) Ao sofrer refração de um meio com índice de refração n1 para outro com índice de refração n2, um raio de luz monocromático tem seu ângulo de incidência reduzido em 5°. Marque a alternativa que traz uma informação incorreta a respeito dessa refração.
(A) Esse raio de luz passou de um meio de menor índice para outro de maior índice de refração.
(B) A alteração no ângulo deve-se a uma diminuição da velocidade de propagação da luz.
(C) O índice de refração relativo entre os meios 1 e 2 é menor que 1.
(D) O índice de refração relativo entre os meios 2 e 1 é menor que 1.
(E) Na passagem de n1 para n2, a velocidade da luz é reduzida.
Resolução:
A redução do ângulo no momento da refração indica que a luz aproximou-se da reta normal, portanto, a refração ocorreu do maior para o menor índice de refração (n1 < n2). O índice de refração relativo entre os meios 2 e 1 é a razão entre os índices de refração n2 e n1. Como o índice do meio 2 é maior, o resultado dessa relação deverá ser um número maior que 1.
Alternativa: D

09) (FPS-PE) Um feixe de luz vermelha é emitido por uma caneta laser (apontador laser) e viaja no ar com uma velocidade de propagação na ordem de 300.000 km/s (c = 3 . 10+8 m/s). Esse feixe de laser passa através de uma peça de acrílico cujo índice de refração vale n = 1,46. A velocidade de propagação do feixe de laser dentro da peça de acrílico será, aproximadamente, de:
(A) 1,5 . 10+6 m/s
(B) 3,0 . 10+8 m/s
(C) 5,0 . 10+7 m/s
(D) 4,0 . 10+8 m/s
(E) 2,0 . 10+8 m/s
Resolução:
A velocidade da luz através do acrílico (vACRIL) pode ser determinada a partir da definição do índice de refração:

Alternativa: E

10) (UFT) Um raio de luz monocromático propaga-se de um meio A para um meio B e forma com a normal, à superfície de separação, ângulos de 30º e 45º, respectivamente. O meio B é o ar, que possui índice de refração igual a 1 e por onde a luz propaga-se com velocidade de 3,0 . 108 m/s. Portanto, a velocidade de propagação da luz no meio A será de: (dados: sen 30º = 1/2; sen 45º = √2/2).
(A) 1,8 . 108 m/s
(B) 2,0 . 108 m/s
(C) √2 . 108 m/s
(D) 1,5√2 . 108 m/s
(E) 3,0√2 . 108 m/s
Resolução:
Por meio da lei de Snell, pode-se determinar o índice de refração do meio A:

A velocidade da luz no meio A pode ser determinada a partir da definição de índice de refração:

Alternativa: D

11) Marque a alternativa correta a respeito do fenômeno da refração da luz.
(A) A refração é caracterizada pela mudança de meio de propagação da luz, que sempre ocasiona aumento em sua velocidade.
(B) O índice de refração é definido como sendo a razão entre a velocidade da luz em um meio qualquer e a velocidade da luz no vácuo.
(C) A lei de Snell só pode ser aplicada quando a refração ocorre entre o ar e um meio qualquer.
(D) Na passagem do maior para o menor índice de refração, a luz sofre aumento em sua velocidade.
(E) Na passagem do menor para o maior índice de refração, a luz sofre aumento em sua velocidade.
Resolução:
Quanto maior for o índice de refração de um meio, menor será a velocidade da luz. Na passagem de um índice maior para outro menor, a velocidade da luz será aumentada.
Alternativa: D

12) Um raio de luz monocromático incide na superfície de separação entre dois meios com índice de refração igual a 1,5 e 2,0, passando do menor para o maior índice. Determine o ângulo de refração sabendo que o ângulo de incidência é de 60°.
Dados: sen60° = 0,87; cos60° = 0,50
(A) 36°
(B) 48°
(C) 41°
(D) 65°
(E) 55°
Resolução:
O ângulo de refração pode ser determinado a partir da lei de Snell.






Alternativa: C

13) A luz amarela se propaga em um determinado vidro com velocidade de 200.000 km/s. Sendo 300.000 km/s a velocidade da luz no vácuo, determine o índice de refração absoluto do vidro para a luz amarela:
(A) n = 1,1
(B) n = 1,2
(C) n = 1,3
(D) n = 1,4
(E) n = 1,5
Resolução:

Alternativa: E
14) Supondo que o diamante apresente índice de refração absoluto 2,41 para a luz amarela, e sendo 300.000 km/s a velocidade da luz no vácuo, calcule a velocidade da luz amarela no diamante.
(A) 100.500 km/s
(B) 124.500 km/s
(C) 136.500 km/s
(D) 148.500 km/s
(E) 152.500 km/s
Resolução:

Alternativa: B

15) (Fatec-SP) Na figura adiante, um raio de luz monocromático se propaga pelo meio A, de índice de refração 2,0. (Dados: sen. 37° = 0,60 sen. 53° = 0,80)
Devemos concluir que o índice de refração do meio B é:
(A) 0,5
(B) 1,0
(C) 1,2
(D) 1,5
(E) 2,0
Resolução:
Os ângulos de incidência e refração são observados em relação à reta Normal, dessa forma o ângulo de incidência é de 37° e o ângulo de refração é de 53°.
De acordo com a lei de Snell, temos:
n1 . seni = n2 . senr
2 . sen 37 = n2 . sen53
2 . 0,6 = n2 . 0,8
n2 = 1,2 / 0,8
n2 = 1,5
Alternativa: D

15) (PUC-MG) Suponha que não houvesse atmosfera na Terra. Nesse caso, é correto afirmar que veríamos:
(A) o Sol nascer mais cedo no horizonte
(B) o Sol se pôr mais cedo no horizonte.
(C) o nascer e o pôr do sol mais tarde.
(D) o nascer e o pôr do sol no mesmo horário como se houvesse atmosfera.
(E) n.d.a
Resolução:
Não existindo atmosfera, veríamos o Sol se pôr mais cedo. Por causa da refração atmosférica, conseguimos ver o Sol por mais tempo quando está se pondo, pois a sua luz sofre desvio na atmosfera.
Alternativa B

16) (UEFS BA) Um raio luminoso incide sobre a superfície de separação entre o ar e o vidro com um ângulo α = 60° e refrata com um ângulo β = 30°, como mostra a figura.
Considerando sen30º = cos60º = 0,5; sen60º = cos30º = 0,87; o índice de refração do ar igual a 1 e o índice de refração do vidro igual a n, então o valor de n é igual a
(A) 1,48
(B) 1,57
(C) 1,63
(D) 1,74
(E) 1,83
Resposta:
A partir da aplicação da lei de Snell-Descartes, podemos definir o índice de refração do vidro.


CASO 3: RESOLVIDAS
01) Analise as proposições a seguir sobre a reflexão da luz:
I – O fenômeno da reflexão ocorre quando a luz incide sobre uma superfície e retorna ao seu meio original;
II – Quando ocorre reflexão difusa, a imagem formada é bastante nítida;
III – Na reflexão regular, os raios de luz propagam-se de forma paralela uns aos outros;
IV – Quando a luz é refletida por uma superfície, o ângulo de reflexão é sempre igual ao ângulo de incidência da luz.
Estão corretas:
(A) I, II e III apenas
(B) I, III e IV apenas
(C) I, II e IV apenas
(D) II, III e IV apenas
(E) todas afirmativas estão corretas
Resolução:
Façamos uma análise de cada uma das proposições:
I – está correta, pois a reflexão ocorre quando a luz incide sobre uma superfície e volta a propagar-se em seu meio de origem;
II – incorreta, pois, quando a reflexão é difusa, os raios de luz retornam ao meio de origem com diferentes direções, de forma que a imagem formada é irregular;
III – correta, pois, na reflexão regular, a luz incide sobre uma superfície lisa e os raios de luz retornam ao seu meio original de propagação paralelos entre si;
IV – Correta, pois, de acordo com as leis da reflexão, o ângulo de reflexão sempre é igual ao de incidência.
Alternativa: B

02) (PUC – SP) O ângulo de incidência, em um espelho plano, é de 30º. Qual o valor do ângulo formado entre o raio refletido e a superfície?
Resolução:
Para calcular o ângulo entre o raio refletido e a superfície, utilizamos a equação:
r = 90 – i
r = 90 – 30
r = 60º
03) (UFB) A propriedade óptica que afirma que o ângulo de incidência é igual ao ângulo de reflexão é válida somente para os espelhos planos?
Resolução:
Não, a regra também é válida para superfícies irregulares e espelhos esféricos.

04) O ângulo entre um raio de luz que incide em uma superfície e o raio de luz refletido por ela é igual a 80º. Qual é o ângulo entre o raio incidente e a reta normal? E qual é o ângulo entre o raio refletido e a superfície?
Resolução:
Para resolver essa questão, observe a figura:
Raio de luz incidente e raio de luz refletido em uma superfície plana
O ângulo de 80º entre o raio incidente e o refletido é o dobro do ângulo entre o raio incidente e a reta normal. Portanto, para encontrar o ângulo de incidência, basta dividi-lo por dois.
i = 80 = 40º
     2

Esse ângulo é igual ao ângulo entre o raio refletido e a reta normal, que é o ângulo de reflexão.
R = 40º
Para calcular o ângulo entre o raio refletido e a superfície, usamos a equação:
θ = 90 – r
θ = 90 – 40
θ = 50º

06) (UFRN) Ana Maria, modelo profissional, costuma fazer ensaios fotográficos e participar de desfiles de moda. Em trabalho recente, ela usou um vestido que apresentava cor vermelha quando iluminado pela luz do sol.
Ana Maria irá desfilar novamente usando o mesmo vestido. Sabendo que a passarela onde ela vai desfilar será iluminada agora com luz monocromática verde, podemos afirmar que o público perceberá seu vestido como sendo:
(A) verde, pois é a cor que incidiu sobre o vestido.
(B) Preto, porque o vestido só reflete a cor vermelha.
(C) De cor entre vermelha e verde devido à mistura das cores.
(D) Vermelho, pois a cor do vestido independe da radiação incidente.
Resolução:
Nesse caso, o vestido é vermelho sob luz solar porque reflete a luz vermelha. Sob a luz verde, aparecerá preto, pois ele não reflete esta luz, ele a absorve.
Alternativa: B

08) (Unirio-RJ) Durante o final da Copa do Mundo, um cinegrafista, desejando alguns efeitos especiais gravou cena em um estúdio completamente escuro, onde existia uma bandeira da “Azzurra” (azul e branca) que foi iluminada por um feixe de luz amarela monocromática. Quando a cena foi exibida ao público, a bandeira apareceu:
(A) verde e branca
(B) verde e amarela
(C) preta e branca
(D) preta e amarela
(E) azul e branca
Resolução:
A região azul, não reflete a luz amarela, com isso aparecerá preta.
Já a região branca, reflete a luz amarela, portanto aparecerá amarela.
Alternativa: D

CASO 5: RESOLVIDAS
01) (UFMG) Qual a alternativa que melhor explica por que a profundidade aparente de uma piscina é menor do que a real?
(A) A luz refletida na superfície da água é perturbada pela luz refletida pelo fundo da piscina;
(B) A luz refletida pela superfície da água sofre refração no ar;
(C) A luz refletida pelo fundo da piscina sofre reflexão total na superfície da água;
(D) A luz refletida pelo fundo da piscina sofre refração ao passar da água para o ar;
(E) A luz é refratada ao passar do ar para a água.
Resolução:
Quando a luz refletida pelo fundo da piscina volta a emergir à superfície, ela sofre refração novamente. Como o índice de refração do ar é menor que o da água, a luz propaga-se com maior velocidade nesse meio, afastando-se da reta normal à superfície. Isso causa a impressão de que o fundo da piscina está mais perto.
Alternativa: D
02) (UFG) Um ponto luminoso encontra-se imerso na água em uma piscina totalmente limpa, quando visto por um observador que esteja fora da piscina (no ar) e que olha com uma inclinação de 45º em relação ao eixo normal da superfície da água. É CORRETO afirmar que:
(A) o ponto luminoso parecerá mais afastado da superfície da água do que realmente está;
(B) o ponto luminoso parecerá mais próximo da superfície da água do que realmente está;
(C) o fato de a luz estar mudando de meio não interfere na percepção visual do observador;
(D) a luz não irá conseguir passar da água para o ar;
(E) o ponto luminoso não será percebido pelo observador.
Resolução:
Como a luz que é observada sai de um meio com maior índice de refração (água) para um índice com menor índice de refração (ar), a refração dos raios de luz fará com que o objeto pareça estar mais próximo da superfície da água.
Alternativa: B

03) Um estudante de Física avista um peixe dentro da água e faz as seguintes afirmações relacionadas à imagem que observa:
I – A imagem do peixe vista por fora d'água é uma imagem virtual, pois é formada pelo prolongamento dos raios de luz;
II – Por causa da refração da luz proveniente do meio líquido, o peixe parece estar em uma profundidade maior;
III – Os raios de luz que emanam do peixe e propagam-se em direção perpendicular à interface da água sofrerão refração, mas sua direção de propagação não será alterada.
Estão corretas:
(A) I e II
(B) somente II
(C) II e III
(D) somente I
(E) I e III
Resolução:
I – Verdadeiro. De fato a imagem vista do peixe é virtual, pois é direita e formada pelo prolongamento dos raios de luz refratados;
II – Falso. A percepção é de que o peixe está mais próximo da superfície da água, pois os raios de luz saem do meio de maior índice de refração para um meio com menor índice de refração, logo, esses raios de luz afastam-se da reta normal, dando a ilusão de o peixe estar mais próximo da superfície e não o contrário, como afirmou o estudante;
III – Verdadeiro. A velocidade dos raios de luz mudará ao sair de dentro da água para o ar, mas como a direção do feixe é perpendicular à superfície, a direção de propagação não sofrerá qualquer desvio lateral.
Alternativa: E

04) Um pescador deixa cair uma lanterna acesa em um lago a 10.0 m de profundidade. No fundo do lago, a lanterna emite um feixe luminoso, formando um pequeno ângulo com a vertical. Considere o índice de refração da água como 1,33 e determine a profundidade aparente (h) vista pelo pescador.
(A) 2,5 m
(B) 5,0 m
(C) 7,5 m
(D) 8,0 m
(E) 9,0 m
Resolução:
A relação usada para os dioptros planos nos diz que:

Tomando os dados fornecidos pelo enunciado do exercício:

Fazendo a divisão do lado esquerdo da equação. temos que:

Por fim, a multiplicação cruzada dos termos fornece um valor de aproximadamente 7,5 m:

Alternativa: C

RESOLVIDOS – FENÔMENOS ÓPTICOS
01) (FMABC SP) As imagens abaixo correspondem a um acessório de segurança para quem tem que transportar um bebê na cadeirinha no banco de trás – o espelho retrovisor para bebê. Para que os pais possam ver o filho sentado na cadeirinha, fixada no banco traseiro e de costas para os ocupantes dos bancos da frente do carro, o espelho deve ser fixado no encosto de cabeça do banco traseiro, defronte ao bebê. Assim, os pais, ao olharem para o espelho retrovisor interno do automóvel, poderão ver a imagem refletida do bebê. O princípio da óptica geométrica que permite que isso seja possível é
(A) refração luminosa.
(B) dispersão luminosa.
(CF) independência dos raios luminosos.
(D) reversibilidade dos raios luminosos.
Resolução:
O princípio óptico envolvido na formação de imagens em espelhos é a reflexão da luz. O caminho da luz feito do bebê até o olho do motorista pode ser executado em qualquer sentido (motorista-bebê ou bebê – motorista), este fenômeno é denominado de reversibilidade da luz.
Alternativa: D

02) (IFSC) Com base nos princípios da óptica geométrica, analise as afirmativas abaixo.
I. Na reflexão, o raio incidente e o raio refletido estão contidos no mesmo plano que a reta normal, portanto são congruentes.
II. Quando a luz incide numa fronteira separadora de dois meios, pode sofrer reflexão, absorção e refração.
III. Ao observarmos uma pessoa através de um espelho plano, também seremos vistos por ela. Este fenômeno é descrito pelo Princípio da Independência dos Raios Luminosos.
IV. A faixa de frequência de ondas capaz de sensibilizar o olho humano é denominada de espectro visível.
V. Podemos considerar que a “sombra” de uma nuvem projetada sobre o solo é do mesmo tamanho da própria nuvem, devido aos raios solares serem aproximadamente paralelos.
É CORRETO afirmar que
(A) apenas II, IV e V são verdadeiras.
(B) apenas II e III são verdadeiras.
(C) apenas III e V são verdadeiras.
(D) apenas I, II, III e VI são verdadeiras.
(E) apenas III e IV são verdadeiras.
Resolução:
I) falsa. Raio incidente, raio refletido e normal são coplanares.
II) verdadeira.
III) falsa. O fenômeno descrito é o da reversibilidade dos raios de luz.
IV) verdadeira.
V) verdadeira.
Alternativa: A

03) Marque a alternativa correta a respeito dos fenômenos da reflexão, refração e absorção da luz.
(A) Um único raio de luz incidente não pode sofrer os três fenômenos ópticos ao mesmo tempo.
(B) As leis da reflexão se aplicam tanto à reflexão regular quanto à reflexão difusa.
(C) As leis da refração só podem ser aplicadas no caso da refração difusa.
(D) A reflexão de um espelho plano é difusa.
(E) Todas as alternativas anteriores estão incorretas.
Resolução:
a) errada. Um único raio de luz pode sofrer, ao mesmo tempo, mais de um fenômeno óptico.
b) errada. As leis da reflexão são aplicadas para a reflexão regular da luz.
c) errada. As leis da refração são aplicadas para a refração regular da luz.
d) errada. A reflexão de um espelho plano é regular.
e) correta.
Alternativa: E

04) Ao observar o asfalto em dias quentes podemos perceber a formação de imagens que aparentam poças d’água.
Marque a alternativa que apresenta o nome dado a este evento e o fenômeno óptico envolvido em sua ocorrência.
(A) Miragens, reflexão da luz
(B) Dispersão, refração da luz
(C) Difração, reflexão da luz
(D) Miragens, refração da luz
(E) Miragens, absorção da luz.
Resolução:
A diferença de temperatura nas camadas de ar próximas ao chão geram alterações no índice de refração do meio, fazendo com que a luz sofra uma curva e produza uma imagem, denominada de miragem.
Alternativa: D

RESOLVIDOS – VELOCIDADE DA LUZ E VELOCIDADE DO SOM
01) Em um período de festa junina, uma jovem observou de longe o lançamento de um foguete. Ao ver a luz, ela marcou um tempo de 3s até que ouviu o estrondo do foguete. Sabendo que a velocidade do som no ar é de 340 m/s, marque a opção que indica a distância entre a jovem e o ponto de lançamento do foguete e o motivo pelo qual a luz foi percebida antes que o som.
(A) 2000 m; a luz possui velocidade infinitamente maior que a velocidade do som, por isso, foi percebida primeiro.
(B) 1020 m; a luz foi percebida primeiro porque a visão é mais sensível a estímulos quando comparada com a audição.
(C) 1020 m; a luz possui velocidade infinitamente maior que a velocidade do som, por isso, foi percebida primeiro.
(D) 1200 m; a luz possui velocidade infinitamente maior que a velocidade do som, por isso, foi percebida primeiro.
(E) 1200 m; a luz possui velocidade infinitamente menor que a velocidade do som, por isso, foi percebida primeiro.
Resolução:
A partir da definição de velocidade média, podemos escrever:
vSOM = Δs
             Δt

vSOM . Δt = Δs
Δs = 340 . 3
Δs = 1020 m
A velocidade da luz é de 300.000.000 m/s, infinitamente superior à velocidade do som, que é de apenas 340 m/s; por isso, ela foi percebida primeiro.
Alternativa: C

02) Um navio possui uma sonda que funciona por meio da produção de ondas sonoras. Ao identificar um obstáculo submerso, o som gerado pela sonda gasta 8 s para ir e retornar até o objeto. Marque a opção que determina a distância entre o navio e o objeto no fundo do mar e explica por que a velocidade do som na água é maior que a velocidade do som no ar.
Dados:
Velocidade do som no ar = 340 m/s
Velocidade do som na água = 1430 m/s
(A) 5700 m; a velocidade do som na água é maior por conta da maior elasticidade da água quando comparada com o ar.
(B) 4500 m; a velocidade do som na água é maior por conta da menor elasticidade da água quando comparada com o ar.
(C) 5720 m; a velocidade do som na água é maior por conta da menor elasticidade da água quando comparada com o ar.
(D) 5720 m; a velocidade do som na água é maior por conta da maior elasticidade da água quando comparada com o ar.
Resolução:
A partir da definição de velocidade média, podemos escrever:
vSOM = Δs
             Δt

vSOM . Δt = Δs
O tempo para ir e voltar até o obstáculo é de 8 s, sendo assim, o tempo a ser considerado nos cálculos deve ser apenas o tempo de ida: 4s.
vSOM . Δt = Δs
1430 . 4 = Δs
Δs = 5720 m
As ondas sonoras transmitem-se por compressão e descompressão do meio, sendo assim, possuem maior velocidade nos meios que apresentam maior facilidade para sofrer as compressões. Nesse quesito, a água é melhor que o ar.
Alternativa: D

03) (UFPE) Diante de uma grande parede vertical, um garoto bate palmas e recebe o eco um segundo depois. Se a velocidade do som no ar é 340 m/s, o garoto pode concluir que a parede está situada a uma distância aproximada de:
(A) 17 m
(B) 34 m
(C) 68 m
(D) 170 m
(E) 340 m
Resolução:
No eco, o som produzido atinge um obstáculo e retorna à fonte, de modo que a distância a ser considerada em cálculo deve ser o dobro da distância entre a fonte sonora e o obstáculo. Sendo assim, temos:
vSOM = 2d
             Δt

vOM . Δt = 2d
2d = 340 . 1
d = 170 m
Alternativa: D
04) Ao ver um raio cortar e iluminar o céu, um garoto marcou o tempo entre o raio e o trovão. Sabendo que o tempo marcado pelo menino foi de 2 s e que a velocidade do som no ar é de 340 m/s, determine a distância entre o garoto e o ponto de queda do raio.
(A) 680 m
(B) 580 m
(C) 1000 m
(D) 1500 m
(E) 650 m
Resolução:
A partir da definição de velocidade média, podemos escrever:
vSOM = Δs
             Δt

vSOM . Δt = Δs
340 . 2 = Δs
Δs = 680 m
Alternativa: A

ESPELHO
É toda superfície que reflete a luz. Os espelhos podem  ser: Espelhos Planos ou Espelho Curvos

Espelhos planos
Imagens formada por espelhos planos
No espelho e atingem nossos olhos. Assim, recebemos raios luminosos que descreveram uma trajetória angular e temos a impressão de que são provenientes de um objeto atrás do espelho.
Características da Imagens formadas por ESPELHOS PLANOS são sempre:
  • Virtual: formada atrás do espelho.
  • Direita: mesma posição do objeto original.
  • Igual ao objeto: mesmo tamanho do objeto original.
  • Distância do objeto ao espelho é igual a distância do espelho a imagem.
  • A velocidade de aproximação e/ou afastamento do objeto ao espelho é igual ao velocidade de aproximação e/ou afastamento da imagem ao espelho.
 Observação: um espelho plano causa a inversão da imagem do sentido (esquerda – direita), originado imagens de letras ao contrário, por exemplo.
Espelhos Esféricos
Espelho côncavo - É aquele cuja superfície espelhada (polida) é a superfície interna da casca esférica, como é o caso dos espelhos de estojos de maquiagem.Espelho convexo - É aquele cuja superfície espelhada (polida) é a superfície externa da casca esférica, como é o caso dos utilizados em alguns tipos de espelhos retrovisores e espelhos utilizados em supermercados e farmácias.  
     

Os principais elementos de um espelho esférico.

Raio de curvatura (R) de um espelho esférico é a medida do raio da casca esférica original do espelho, ou seja, representa a distância do centro de curvatura até o vértice do espelho.Centro de curvatura  (C) coincide com o centro da casca esférica que originou o espelho.Foco (F) é o ponto médio do segmento que une o centro de curvatura e o vértice e é por onde são refletidos a maior parte dos raios.Distância focal  (f) é a medida da distância entre o foco e o vértice. Como o foco está situado no ponto médio do eixo centro – vértice, pode-se afirmar que a sua medida é a metade da medida do raio de curvatura.Vértice (V) é o ponto tangencial à circunferência do espelho que marca a interseção entre  o espelho e o eixo do mesmo.Eixo do espelho  (e) é a linha de centro que une o foco, o centro de curvatura e o vértice do espelho. 
Formação de imagens formada por espelhos CURVOSAo contrário dos espelhos planos, os espelhos esféricos formam imagens de tamanhos diferentes do tamanho do objeto. Enquanto o espelho convexo forma imagens sempre menores que o objeto, o espelho côncavo forma imagens de diferente tamanhos, dependendo da posição em que o objeto é colocada sobre o seu eixo.Espelhos Convexos - Fornece apenas um tipo: Imagem virtual, direita e menor que o objeto.Para determinarmos como são formadas as imagens em um espelho convexo, devemos conhecer o comportamento dos raios de luz incidentes, ou seja, quando atingem a superfície do espelho e refletem as imagens. 
Espelhos Côncavos - Nestes espelhos, o tipo de imagem Depende da posição na qual o objeto é colocado que pode ser: em relação ao centrofoco vértice do espelho. Ou seja, fornece 5 tipos de imagens.Vejamos cada caso:1) Objeto real - objeto colocado antes do centro de curvatura: Imagem real, invertida e menor que o objeto.
2) Objeto real - objeto colocado no centro de curvatura: Imagem real, invertida e do mesmo tamanho do objeto.3) Objeto real - objeto colocado entre o centro de curvatura e o foco: Imagem real, invertida e maior que o objeto.4) Objeto real - objeto colocado no foco: Imagem imprópria, ou seja, localizada no infinito. Ou seja, não forma imagem.5) Objeto real - objeto colocado entre o foco e o vértice: Imagem virtual (atrás do espelho), direita e maior que o objeto.

Equação de Gauus - Um objeto pode ser real ou virtual. No caso dos espelhos, dizemos que o objeto é virtual se ele se encontra “atrás” do espelho. No caso de espelhos esféricos a imagem de um objeto pode ser maior, menor ou igual ao tamanho do objeto.
A imagem pode ainda aparecer invertida em relação ao objeto. Se não houver sua inversão dizemos que ela é direita. Equação de Gauss
SOMBRA E PENUMBRA
Formação de Penumbra e Sombra:

Comportamento de um raio de luz no interior de uma fibra óptica:
 Numero de imagens formadas por dois espelhos planos.      
n = número de imagens.α = ângulo entre os espelhos LENTESSão objetos comuns utilizados em óculos, projetores, máquinas fotográficas e de filmar, etc.
São portanto muito úteis e é importante conhecer o seu funcionamento. Uma lente provoca uma mudança de direção nos raios de luz que nela incidem. 
As Lentes Esféricas podem ser de dois tipos:São dispositivos ópticos que funcionam por refração da luz e são muito utilizadas no nosso dia a dia, como nos óculos, nas lupas, nas câmeras fotográficas, nas filmadoras e em telescópios.
O material que as constitui normalmente é o vidro, mas o plástico também pode ser utilizado. As principais características desses dispositivos são a transparência e a superfície esférica.De acordo com a curvatura apresentada, as lentes esféricas podem ser classificadas como:Lentes DivergentesNestas lentes, os raios de luz incidem paralelos ao eixo principal, eles sofrem dupla refração e se espalham.
Como o foco dessas lentes é formado pelo encontro de projeções dos raios de luz incidentes, ele é classificado como virtual.
Lentes ConvergentesNestas lentes, os raios de luz incidem paralelos ao eixo principal e, após sofrerem refração, se concentram em um único ponto, este ponto é o foco.
O foco das lentes convergentes é classificado como foco real, pois é fruto do encontro dos raios de luz refratados.Elementos das lentes esféricasO que caracteriza uma lente esférica são os seus elementos geométricos, que são:C1 e C2: centros de curvatura das faces esféricas;R1 e R2: raios de curvatura das faces esféricas;Eixo principal da lente: onde estão contidos C1 e V1;e: espessura da lente;V1 e V2: Vértices da lente. 

Dioptria
É uma unidade de medida que se refere ao poder de refração das lentes em um sistema óptico, ou seja, o famoso “grau” dos óculos popularmente conhecido.

Assim temos que 1 grau é igual a 1 dioptria.Para entendermos melhor como é calculado a dioptria é interessante conhecermos um pouco de curvaturas. Segue abaixo na figura 1 três ilustrações que nos ajudarão a compreender: Ilustração de um segmento de reta, curva e circunferência de raio (R).
Visivelmente percebemos que a diferença entre o segmento de reta (AB) e a curva (C), é justamente a curvatura, ou seja, para o segmento de reta (AB) a curvatura é igual a zero e a curvatura da curva (C) é diferente de zero.
Onde R é o valor do raio da circunferência e C representa o valor da curvatura. Podemos concluir então que a curvatura de uma curva é definida como o inverso do raio, assim temos que o valor de curvatura é menor que 1 e quanto maior for o raio será menor a curvatura.
O calculo da dioptria é muito parecido com o da curvatura, no entanto, apenas alteramos o raio da equação (1) para a distância focal (f), assim teremos que a dioptria será o inverso da distância focal da lente, para esse calculo temos que a unidade de medida da dioptria é o inverso do metro (m-1).
Assim podemos dizer então que, uma lente convergente de distância focal igual a 1 m, terá a potência de 1 dioptria. Se a distância focal for de 0,5 m, a potência será de 2 dioptrias.O olho de um ser humano tem a distância focal de aproximadamente 17 mm, dependo da deficiência na visão de uma pessoa, existe a necessidade de uma maior ou menor distância focal para que os raios luminosos possam convergir sobre a retina, essa correção é feita com a utilização das lentes.   Imagem formada por lentes esféricas
  Imagens das lentes convergentes
As lentes esféricas, as convergentes formam 5 tipos distintos de imagens:

A - Quando o objeto é posicionado antes do ponto antiprincipal, a lente forma uma imagem é:
- Real
- Invertida e menor que o objeto.

Exemplo: Máquina fotográfica e olho humano.Objeto antes do ponto antiprincipalB - Quando o objeto  é posicionado sobre o ponto antiprincipal, a lente forma uma imagem é: 
- Real
- Invertida e igual ao objeto.

Exemplo: Máquinas de fotocópiaObjeto sobre o ponto antiprincipalC - Quando o objeto  é posicionado entre o ponto antiprincipal e o foco da lente, a imagem formada é:
- Real
Invertida e maior que o objeto.

Exemplo: ProjetoresObjeto entre o ponto antiprincipal e o focoD - Quando o objeto é posicionado sobre o foco da lente, não forma imagem, pois os raios refratados são paralelos e nunca se cruzam para formar uma imagem do objeto.Objeto sobre o focoE - Quando o objeto é posicionado entre o foco e o centro óptico da lente, sua imagem  é:
- Virtual
- Direita e maior que o objeto.

Exemplo de equipamento: Lupas.Objeto entre o foco e o centro ópticoImagem da lente divergente
As lentes divergentes são capazes de formar apenas um tipo de imagem, pois, qualquer que seja a posição de um corpo diante de uma lente divergente, sua imagem é:
- Virtual
- Direita e menor que o objeto

Exemplo de utilização: 

Lentes desse tipo são utilizadas para a correção da miopia. Imagem da lente divergenteLentes de contato
São dispositivos médicos colocados diretamente na superfície dos olhos para corrigir a visão. Elas podem corrigir problemas de visão à longa ou curta distância

Miopia
Hipermetropia
Astigmatismo

Os problemas de visão para perto relacionados à idade (presbiopia).

O princípio das lentes de contato é o mesmo dos óculos.

Elas desviam a luz e a redirecionam para sua retina para dar a você uma visão mais nítida.

Do que são feitas as lentes de contato?
Hoje em dia, a maioria das pessoas usa lentes de contato gelatinosas, feitas, a princípio, de um material rico em umidade chamado hidrogel. Estas lentes permitem que o oxigênio atravesse sua superfície e chegue até seus olhos, mantendo-os saudáveis e com sensação de frescor. 
Existem diferentes tipos de lentes de contato disponíveis que se adequam a sua visão e estilo de vida. Seu oftalmologista pode ajudá-lo a escolher o tipo certo para você. 
POR QUE VEMOS OS OBJETOS?
Imagem formada no olho humano
Quando olhamos na direção de algum objeto, a imagem atravessa a córnea e chega à íris, que regula a quantidade de luz recebida por meio de uma abertura chamada pupila.
Quanto maior a pupila, mais luz entra no olho.
Passada a pupila, a imagem chega ao cristalino, e é focada sobre a retina. A lente do olho produz uma imagem invertida, e o cérebro a converte para a posição correta.
Na retina, mais de cem milhões de células fotorreceptoras transformam as ondas luminosas em impulsos eletroquímicos, que são decodificados pelo cérebro. 
CÂMARA ESCURA
Foi a primeira grande descoberta da fotografia.
É uma caixa composta por paredes opacas, que possui um orifício em um dos lados, e na parede paralela a este orifício, uma superfície fotossensível é colocada.
O funcionamento da câmara escura é de natureza física. O princípio da propagação retilínea da luz permite que os raios luminosos que atingem o objeto e passem pelo orifício da câmara sejam projetados no anteparo fotossensível na parede paralela ao orifício. Esta projeção produz uma imagem real invertida do objeto na superfície fotossensível.
Quanto menor o orifício, mais nítida é a imagem formada, pois a incidência de raios luminosos vindos de outras direções é bem menor. 

Máquina fotográfica
Inspirado no funcionamento do olho o homem criou a máquina fotográfica. Portanto, em nossos olhos a córnea funciona como a lente da câmera, permitindo a entrada de luz no olho e a formação da imagem na retina. 
No século XIV já se aconselhava o uso da Câmara Obscura como auxílio ao desenho e à pintura. Leonardo da Vinci (1452-1519) fez uma descrição da Câmara
 
CASO 1: RESOLVIDOS
01) Um raio de luz incide no ponto I um espelho plano e, após a reflexão, passa pelo ponto P. Determine o ângulo de incidência:
Resolução:
A linha tracejada corresponde ao raio de luz incidente.h² = 2² + 2²h² = 4 + 4h² = 8h = √8h = 2√2 msenθ = 2/2.√2 = 1/√2senθ = (1/√2).( √2/√2)senθ = (√2/2)θ = arcsen(√2/2) = 45°
A reta normal é perpendicular ao espelho, logo: θ + β = 90°45º + β = 90ºβ = 90° - 45°β = 45° 

02) Um raio de luz incide sobre um espelho plano. De acordo com as condições dadas na figura, determine o valor do ângulo de incidência.Resolução:
Os ângulos opostos pelo vértice formados entre retas concorrentes são iguais, logo, temos que:Impondo a normal ao esquema, determinaremos o ângulo de incidência θ. 40 + θ = 90θ = 90 – 40θ = 50º 

03) Um espelho plano desloca-se com velocidade de 10m/s em módulo. Considere que o espelho esteja se afastando de uma pessoa e ela esteja de frente para ele. Determine o módulo da velocidade da imagem da pessoa em relação ao solo e em relação ao espelho.

Resolução:
Quando o referencial é considerado o solo, temos que a velocidade de afastamento da imagem equivale ao dobro da velocidade do espelho.Logo vi = 2.ve = 2.10 = 20m/sConsiderando o próprio espelho como referencial, a velocidade da imagem é igual à velocidade do espelho. Logo vi = ve = 10m/s 

04)
Um raio de luz incide em um espelho plano. Gira-se o espelho de um ângulo α em torno de um eixo perpendicular ao espelho e perpendicular ao plano de incidência. O ângulo formado pelos raios refletidos antes e após a rotação é de 40°.

Determine o valor do ângulo α.
Resolução:
Através de relações geométricas temos que: Δ = 2.α40 = 2.αα  = 40/2α = 20° 

05) Dois espelhos são alinhados de forma que as direções normais de cada uma de suas superfícies formam um ângulo α entre si, como mostra a figura abaixo:Assinale a alternativa correta relacionada à situação descrita:(A) o número de imagens formadas não depende do ângulo α.(B) o número de imagens formadas é diretamente proporcional ao ângulo α.(C) o número de imagens formadas é inversamente proporcional ao ângulo α.(D) o número de imagens formadas só depende do campo visual de cada espelho.
Resolução:
O número de imagens conjugadas por uma associação de espelhos planos pode ser calculada por meio da seguinte relação:
Observando a equação acima, é possível perceber que, quanto maior for o ângulo α, menor será o valor de N, isso é, menos imagens serão formadas. Portanto, é correto dizer que o número de imagens e o ângulo entre os espelhos são inversamente proporcionais.
Alternativa: C 

06) Dois espelhos planos são alinhados de modo que o ângulo que se forma entre eles é de 90º. O número de imagens formadas pela associação desses espelhos é igual a:
(A) 4
(B) 3
(C) 2
(D) 5
(E) 7
Resolução:
Primeiramente, devemos usar a equação que calcula o número de imagens conjugadas pela reflexão dos espelhos associados:

Para o ângulo α = 90º, teremos o seguinte cálculo a ser resolvido:

Portanto, quando o ângulo de associação entre os espelhos é igual a 90º, são formadas 3 imagens.Alternativa: B 

07) (CEFET-CE) Observando as imagens formadas por dois espelhos planos de um objeto entre eles colocado, Syned, um curioso aluno, verifica que, para determinado ângulo, formam-se 5 imagens. Entretanto, fazendo variar o ângulo entre os espelhos, o número de imagens diminui. Pode-se concluir que:
(A) o ângulo era inicialmente de 60°, e o ângulo entre os espelhos estava aumentando.
(B) o ângulo era inicialmente de 30°, e o ângulo entre os espelhos estava aumentando.
(C) o ângulo era inicialmente de 60°, e o ângulo entre os espelhos estava diminuindo.
(D) o ângulo era inicialmente de 72°, e o ângulo entre os espelhos estava diminuindo.
(E) o ângulo era inicialmente de 72°, e o ângulo entre os espelhos estava aumentando.
Resolução:
Vamos calcular qual o ângulo em que os espelhos encontravam-se antes de o aluno girá-los:

O número de imagens era igual a 5, portanto teremos que:

No cálculo executado acima, multiplicamos os dois lados da equação por α para facilitar a resolução. Obtivemos um ângulo igual a 60º. Como o número de imagens formadas pelos espelhos passou a diminuir, conclui-se que o ângulo entre os espelhos estava aumentando.Alternativa: A 

08) Quando dois espelhos planos são associados com um ângulo igual a 30º, formam-se neles
(A) 6 imagens
(B) 14 imagens
(C) 12 imagens
(D) 11 imagens(E) não é possível calcular o número de imagens formadas.
Resolução:
De acordo com o ângulo formado entre os dois espelhos, podemos calcular, por meio da equação abaixo, o número de imagens formadas entre eles:Com o ângulo α = 30º, teremos a seguinte resolução:

Assim, quando dois espelhos planos são associados com um ângulo igual a 30º, formam-se 11 imagens.Alternativa: D  

CASO 3: RESOLVIDOS
01) Um comerciante deseja instalar um espelho esférico que lhe forneça um grande campo visual de seu comércio a fim de monitorá-lo mais eficientemente. O tipo de espelho mais indicado para tal fim é:
(A) um espelho plano.
(B) um espelho esférico côncavo.
(C) um espelho esférico convexo.
(D) um espelho parabólico.
Resolução:
Os espelhos esféricos convexos são conhecidos por produzir imagens direitas e reduzidas. Dessa forma, seu campo visual é maior, podendo mostrar mais detalhes em sua imagem. Portanto, o espelho esférico convexo é o mais indicado para tal aplicação.Alternativa: C 
02) Um estudante de Física dispõe-se de uma grande quantidade de espelhos esféricos distintos. Durante uma aula prática, o seu professor pediu para que ele construísse um dispositivo capaz de captar a luz do Sol a fim de aquecer uma pequena panela. Esse aluno deve escolher um espelho:
(A) convexo.
(B) côncavo.
(C) esférico.
(D) parabólico.
(E) plano.
Resolução:
O único espelho esférico da lista de opções capaz de concentrar os raios de luz em um ponto é o espelho côncavo. Como o Sol é uma fonte de luz muito distante, os seus raios de luz incidem na superfície do espelho quase paralelamente ao seu eixo de simetria, sendo assim, são refletidos na direção do seu foco, cruzando-se. A concentração de raios de luz nesse ponto é capaz de aquecer a panela graças à grande quantidade de energia que é transmitida por esses raios.Alternativa: B 

03) Motivado a enxergar mais carros que se aproximem da traseira do seu veículo, um motorista resolveu instalar um espelho esférico em um de seus retrovisores. Para sua surpresa, ele obteve somente imagens invertidas dos carros distantes. Qual foi o tipo de espelho escolhido pelo motorista e qual espelho seria a escolha correta?
(A) plano e esférico, respectivamente
(B) convexo e côncavo, respectivamente
(C) côncavo e convexo, respectivamente
(D) convexo e parabólico, respectivamente
Resolução:
O motorista instalou um espelho côncavo em seu retrovisor, uma vez que todos os objetos posicionados após a distância focal desse tipo de espelho formam imagens reais e invertidas. A escolha correta, por sua vez, seria um espelho convexo, capaz de formar imagens virtuais e reduzidas, com maior campo visual.Alternativa: C 

04) A dona de uma ótica resolveu comprar novos espelhos para que os seus clientes pudessem ver mais detalhes das armações vendidas na loja por meio de imagens ampliadas. O tipo de espelho escolhido pela empresária foi:
(A) plano.
(B) esférico.
(C) côncavos.
(D) convexo.
Resolução:
Quando um objeto é colocado entre o foco e o vértice de um espelho côncavo, o espelho é capaz de conjugar uma imagem virtual, direita e ampliada desse objeto, portanto, essa é a única escolha possível para o fim descrito no enunciado do exercício.
Alternativa: C  

OLHO HUMANO & LENTES
Oi, este olhar tem zap? rsrsrs

COMPONENTES DO OLHO HUMANO

Cílios
São pelos localizados na borda da pálpebra e servem para proteger o olho de materiais em suspensão no ar, como a poeira.
Conjuntiva
Membrana transparente que reveste a parte anterior do olho e a superfície interior das pálpebras.

Córnea
É o tecido transparente que cobre a pupila, a abertura da íris. Junto com o cristalino, a córnea ajusta o foco da imagem no olho.

Coróide
Camada média do globo ocular. Constituída por uma rede de vasos sangüíneos, ela supre a retina de oxigênio e outros nutrientes.


Corpo Ciliar
Localizado atrás da íris o corpo ciliar é responsável pela formação do humor aquoso e pela acomodação, ou seja, mobilidade do cristalino.

Cristalino
Lente transparente e flexível, localizada atrás da pupila. Funciona como uma lente, cujo formato pode ser ajustado para focar objetos em diferentes distâncias, num mecanismo chamado acomodação.

Esclera
Camada externa do globo ocular – parte branca do olho. Semi-rígida, ela dá ao globo ocular seu formato e protege as camadas internas mais delicadas.


Fóvea Central
Porção de cada um dos olhos que permite perceber detalhes dos objetos observados. Localizada no centro da retina, é muito bem irrigada de sangue e possibilita, através das células cônicas, a percepção das cores.

Humor Aquoso
Líquido transparente que preenche o espaço entre a córnea e o cristalino, sua principal função é nutrir estas partes do olho e regular a pressão interna.

Humor Vítreo
Líquido que ocupa o espaço entre o cristalino e a retina
Íris - É um fino tecido muscular que tem, no centro, uma abertura circular ajustável chamada de pupila

Mácula Lútea

Ponto central da retina. É a região que distingue detalhes no meio do campo visual.

Músculos Ciliares
Ajustam a forma do cristalino. Com o envelhecimento eles perdem sua elasticidade, dificultando a focagem dos objetos próximos e provocando presbiopia.

Músculos Extrínsecos
Conjunto de seis músculos responsáveis pelo movimento dos olhos. Trabalham em sincronismo, entre si, propiciando a movimentação simultânea dos olhos. Caso ocorra alguma alteração neste sincronismo teremos a deficiência ocular chamada estrabismo.

Nervo Óptico
É a estrutura formada pelos prolongamentos das células nervosas que formam a retina. Transmite a imagem capturada pela retina para o cérebro.


Pálpebras
Consideradas anexos oculares, tem como função proteger o olho na sua parte mais anterior.

Através da sua movimentação (piscar), espalha a lágrima produzida pelas glândulas lacrimais, umedecendo e nutrindo a córnea e retirando substâncias e stranhas que tenham alcançado o olho

Pupila
Controla a entrada de luz: dilata-se em ambiente com pouca claridade e estreita-se quando a iluminação é maior. Esses ajustes permitem que a pessoa enxergue bem à noite e evitam danos à retina quando a luz é mais forte.

Retina
ua função é receber ondas de luz e convertê-las em impulsos nervosos, que são transformados em percepções visuais.

Texto: Conselho Brasileiro de Oftalmologia
Site: www.cbo.com.br

DEFEITOS DA VISÃO

Presbiopia (“vista cansada”) - A capacida­de do cristalino de alterar seu poder de refração (aco­modação) tende a diminuir com a idade, à medida que ele perde sua elasticidade. Essa alteração caracteriza a presbiopia, que atinge a maioria das pessoas com mais de quarenta anos.
A focalização de objetos próximos fica difícil e pode ser corrigida com o uso de lentes conver­gentes.Hipermetropia
Problema decorrente do fato de o diâmetro do globo ocular ser pequeno demais ou de o sistema de lentes ter pouca refração. Em ambos os casos, os raios luminosos não são desviados o suficiente para que sejam focalizados sobre a retina (teo­ricamente, a imagem se formaria em um ponto depois da retina). Quando o objeto está longe, a pessoa ainda pode focalizá-lo usando seu poder de acomodação (aumentando a convergência do cristalino). Para objetos próximos é necessário um grande aumento de refração, acima da capacidade de acomodação do olho. Consequentemente, o hipermetrope enxerga mal de perto.
A correção da visão é feita com lentes convergentes, que desviam os raios luminosos de maneira que eles se aproximem, isto é, convirjam.

Miopia
Em geral, é causada por um globo ocular muito alongado ou por grande poder de refração do sistema de lentes. Os raios luminosos são focalizados em um ponto antes da retina, onde, se hou­vesse um anteparo, seria formada uma imagem. Quando o objeto está perto, a acomodação ainda con­segue resolver o problema, mas, à medida que a distân­cia aumenta, o cristalino não pode diminuir mais a sua convergência, e o míope passa a ter dificuldade para enxergar de longe.
A correção da visão é feita com lentes divergentes, que desviam os raios luminosos de maneira que eles se afastem uns dos outros, formando um feixe divergente.
Astigmatismo
É consequência de um formato irregular da córnea ou do cristalino, que desviam os raios luminosos de maneira diferente e a imagem fica fora de foco em algumas direções.
A cor­reção é feita com lentes cilíndricas que tenham curvaturas desiguais e compensem a curvatura desigual do olho. Daltonismo
De origem genética, afeta os cones, e a pessoa não consegue distinguir certas cores.
A mais comum é a dificuldade de distinguir o vermelho e o verde, provocada pela ausência dos cones responsáveis por essas cores.
Oservação: se você, daltônico, não consegue diferenciar um certo vermelho de um marrom, mas vê claramente a diferença entre um triângulo e dois triângulos com um risco ao ... 
Estrabismo - Processo também chamado de vesguice, no qual uma alteração da musculatura do olho afeta a visão binocular normal e a noção de tridimensionalidade.
Pode ser corrigido com o uso de óculos ou por cirurgia.DOENÇAS DA VISÃO
Glaucoma
Problema em que o humor aquoso se acu­mula, o que provoca aumento da pressão intra-ocular, que pode lesionar o nervo óptico. Por isso, se a doença não for diagnosticada (a medida da pressão intra-ocular feita pelo oftalmologista é usada para identificar indí­cios de glaucoma) e tratada a tempo (com medicamen­tos ou cirurgia), poderá provocar cegueira irreversível (por destruição do nervo óptico). Catarata - Doença em que o cristalino perde parte da transparência, o que dificulta a visão. É mais comum após os cinquenta anos. Pode ser corrigida por meio de cirurgia, na qual se retira o núcleo do cristalino e se coloca em seu lugar uma lente artificial. 
Conjuntivite - Trata-se de uma inflamação da conjuntiva, causada por bactérias, vírus, etc. Os olhos costumam ficar avermelhados, e a pessoa pode apresentar uma sensação incômoda, como se tivesse “areia” nos olhos. É importante procurar o médico e não usar colí­rios ou outros medicamentos por conta própria, uma vez que eles podem provocar efeitos indesejados e até outras doenças.
CASO 1: RESOLVIDOS01) (UFRR) “Podemos definir o defeito visual hipermetropia como sendo um defeito oposto ao defeito visual miopia (não permite visão nítida de um objeto distante). A hipermetropia caracteriza-se por um achatamento do olho na direção do eixo anteroposterior ou por uma convergência diminuída em relação ao olho normal. No caso da hipermetropia, a imagem é formada depois da retina e isso provoca falta de nitidez na formação de imagens próximas.Para que uma pessoa hipermétrope consiga enxergar com nitidez os objetos que estão próximos a ela, é necessário aumentar a convergência de seu olho.
Nesse caso, pode-se utilizar
(A) lentes biconvexas;
(B) lentes planas e paralelas;
(C) espelhos convexos;
(D) lentes bicôncavas;
(E) lentes plano-côncavas.
Resolução:
A correção da hipermetropia é feita a partir do uso de lentes esféricas convergentes, que possuem a capacidade de focalizar a luz em um ponto único. As lentes biconvexas são convergentes e, por isso, podem ser utilizadas na correção desse defeito de visão.
Alternativa: A 

02) (IF-GO) Um olho, em condições normais de funcionamento, consegue conjugar de um objeto real uma imagem nítida, real, invertida e menor que o objeto sobre a retina. A Figura 01 mostra a imagem de uma pessoa e as respectivas lentes que serão utilizadas para a confecção de óculos para corrigir suas ametropias. Na Figura 02, a imagem I é obtida com a utilização correta das lentes de correção, a imagem II e a imagem III representam reproduções das visões dos olhos sem a utilização das lentes.Entre as afirmativas abaixo, marque a que representa a relação correta.
(A) O olho esquerdo sofre de hipermetropia, e o olho direito, de miopia. A reprodução da visão desses olhos sem correção é respectivamente as imagens II e III.
(B) O olho esquerdo sofre de miopia, e o olho direito, de hipermetropia. A reprodução da visão desses olhos sem correção é respectivamente as imagens II e III.
(C) A imagem III é a reprodução da visão de uma pessoa hipermétrope sem correção, e a lente que deve ser utilizada para sua devida correção é a disposta no olho direito.
(D) A lente utilizada para a correção visual do olho esquerdo da pessoa é divergente, e a imagem vista por ele sem a lente de correção é a II.
(E) O olho direito da pessoa sofre de miopia, e a imagem vista por ele sem a lente de correção é a II.
Resolução:
A lente utilizada no olho direito gera uma imagem direita e menor do olho, o que caracteriza a imagem como virtual e a lente como divergente. Lentes divergentes são utilizadas para a correção da miopia. A representação da visão do míope está na imagem II.A lente utilizada no olho esquerdo gera uma imagem direita e maior que o objeto, o que caracteriza a imagem como virtual e a lente como convergente. Lentes convergentes são utilizadas para a correção da hipermetropia. A representação da visão hipermétrope está na figura 2, imagem III.Alternativa: E 

03) Pedro é o filho mais novo de Renata. O garoto reclama a alguns dias de que não consegue enxergar o que sua professora escreve no quadro-negro, mesmo que ele se sente na primeira carteira. Ao levar seu filho ao oftalmologista, Renata teve a notícia de que o garoto tinha dificuldade de enxergar de perto. Assinale a alternativa que contém o nome do problema de visão e o tipo de lente que vai ajudar Pedro.
(A) Hipermetropia, lente esférica
(B Presbiopia, lente convergente
(C) Miopia, lente convergente
(D) Hipermetropia, lente convergente
(E) Estrabismo, lente cilíndrica
Resolução:
A dificuldade de enxergar de perto é uma característica da hipermetropia, tipo de problema de visão que é corrigido com lentes esféricas convergentes.
Alternativa: D 

04) Ao receber a receita de um óculos, um paciente leu a seguinte informação:OD: - 1,0 diOE: + 1,5 diMarque a alternativa correta a respeito das informações dadas na receita.
(A) No olho direito (OD), o paciente tem hipermetropia, por isso, deve utilizar lentes cilíndricas.
(B) No olho esquerdo (OE), o paciente tem hipermetropia, por isso, deve utilizar uma lente convergente.
(C) No olho direito (OD), o paciente tem estrabismo, por isso, deve utilizar uma lente com vergência negativa.
(D) O símbolo “di” significa dioptria e determina o tamanho do foco da lente.
(E) No olho esquerdo (OE), o paciente tem presbiopia, por isso, deve utilizar uma lente esférica.
Resolução:
A vergência (V) é definida pelo inverso do foco e é a característica das lentes esféricas que determina seu poder de desviar a luz incidente. O foco das lentes convergentes é positivo, por isso, sempre resulta em um valor de vergência positivo. O foco das lentes divergentes, por sua vez, é negativo e sempre resulta em um valor de vergência negativo. A dioptria (di) é a unidade de medida da vergência de uma lente.No olho direito (OD), portanto, o paciente deve utilizar uma lente esférica divergente (V < 0), o que indica que esse olho possui miopia. No olho esquerdo (OE), o paciente deve utilizar uma lente esférica convergente (V > 0), o que indica que esse olho possui hipermetropia.
Alternativa: B 

05) (UFG-GO) Ao realizar exames oftalmológicos em um ambiente pouco iluminado, os jovens J1 e J2 descobriram que o diâmetro do corpo vítreo de seus olhos é de 18 mm. Nesse exame, descobriu-se que, para J1, as imagens dos objetos são formadas 13 mm após o cristalino, enquanto que, para J2, o diagnóstico atestou que ele não visualiza nitidamente objetos a 25 cm do olho. Conforme o exposto, quais são, respectivamente, os tipos de lentes corretivas que J1 e J2 devem utilizar e quais células responderam mais eficientemente ao estímulo luminoso?
(A) Divergente e convergente, e bastonetes na córnea.
(B) Divergente e divergente, e bastonetes na retina.
(C) Convergente e divergente, e cones na córnea.
(D) Divergente e divergente, e cones na retina.
(E) Divergente e convergente, e bastonetes na retina.
Resolução:O jovem J1 possui miopia, pois as imagens estão sendo formadas antes da retina, e a correção para esse problema de visão deve ser feita com lentes divergentes. O jovem J2 possui hipermetropia, pois ele não consegue enxergar objetos próximos ao olho. A correção, nesse caso, é feita com lentes convergentes. As células que correspondem aos estímulos luminosos são os bastonetes na retina.
Alternativa: E 

06) (UCB-DF) Certo paciente recebe uma receita do oftalmologista para correção visual determinando uma lente de –4,00 dioptrias para ambos os olhos. Essa situação hipotética indica que as lentes apresentadas são:
(A) divergentes, com distância focal de –0,25 m e, provavelmente, o paciente é míope.
(B) convergentes, com distância focal de 0,40 m e, provavelmente, o paciente é hipermetrope.
(C) divergentes, com distância focal de 1,25 m e, provavelmente, o paciente possui astigmatismo.
(D) convergentes, com distância focal de 0,75 m e, provavelmente, o paciente é hipermetrope.
(E) divergentes, com distância focal de –0,40 m e, provavelmente, o paciente é míope.
Resolução:
Ao observar o valor negativo da vergência da lente (- 4,0 dioptrias), deve-se entender que se trata de uma lente divergente, pois esse tipo possui foco negativo. O problema de visão que é corrigido com uso dessas lentes é a miopia, logo, existe a possibilidade do paciente ser míope.A vergência (V) é o inverso do foco (F), e o foco é o inverso da vergência, assim, temos:F = V – 1 = 4 – 1 = ¼ = 0,25 m O sinal negativo refere-se ao fato de a lente ser divergente.Alternativa: A 

07) Um paciente, ao receber a receita de seus óculos, viu as seguintes informações:OD : 2,5 diOE: - 2,0 diO problema de visão que provavelmente esse paciente possui em cada olho é:
(A) Miopia no olho direito (OD) e astigmatismo no olho esquerdo (OE).
(B) Estrabismo no olho direito (OD) e astigmatismo no olho esquerdo (OE).
(C) Glaucoma nos dois olhos.
(D) Hipermetropia no olho direito (OD) e miopia no olho esquerdo (OE).
(E) Hipermetropia no olho direito (OD) e astigmatismo no olho esquerdo (OE).Resolução:Os números indicados na receita correspondem à vergência das lentes do óculos. A vergência positiva indica que a lente é do tipo convergente, utilizada para corrigir a hipermetropia. A vergência negativa indica que a lente é do tipo divergente, a qual é utilizada para corrigir a miopia.
Alternativa: D 

08) Marque a alternativa correta a respeito dos problemas de visão.
(A) A miopia caracteriza-se pela dificuldade em enxergar de perto, e é corrigida por meio do uso de lentes convergentes.
(B) A hipermetropia caracteriza-se pela dificuldade em enxergar de perto, e é corrigida pelo uso de lentes divergentes.
(C) O glaucoma é caracterizado pela opacidade do cristalino.
(D) O astigmatismo pode ser corrigido com o uso de lentes prismáticas.
(E) A miopia caracteriza-se pela dificuldade em enxergar de longe, e é corrigida por meio do uso de lentes divergentes.
Resolução:
A) Errado - A miopia caracteriza-se pela dificuldade em enxergar de longe, e é corrigida por meio do uso de lentes divergentes.B) Errado - A hipermetropia caracteriza-se pela dificuldade em enxergar de perto, e é corrigida pelo uso de lentes convergentes.
C) Errado - A opacidade do cristalino é causada pela catarata. O glaucoma provoca danos ao nervo óptico.
D) Errado - A correção do astigmatismo é feita com o uso de lentes cilíndricas.
E) Correto.
Alternativa: E  

CASO 3: RESOLVIDO01) (PUC-MG) Na formação das imagens na retina da visão humana, tendo em vista uma pessoa com boa saúde visual, o cristalino funciona como uma lente:
(A) convergente, formando imagens reais, invertidas e diminuídas.
(B) convergente, formando imagens reais, direitas e diminuídas.
(C) divergente, formando imagens virtuais, invertidas e diminuídas.
(D) divergente, formando imagens reais, direitas e diminuídas.
(E) divergente, formando imagens reais, invertidas e de mesmo tamanho.
Resolução:
A imagem formada pelo cristalino sobre a retina é real, invertida (de cabeça para baixo) e menor que o objeto. Essa imagem é “recebida”, invertida e ampliada pelo cérebro para que possamos perceber perfeitamente o mundo ao nosso redor.
Alternativa:  

02) Calcule o aumento linear transversal de uma lente que forma uma imagem de 40 cm de um objeto que possui 10 cm de altura.Resolução:Dados:o = 10 cmi = 40 cmUtilizamos a seguinte equação:A =  i       oSubstituindo os dados, temos:A = 40 / 10A = 4Isso significa que a imagem é quatro vezes maior que o objeto. 03) Sobre as lentes esféricas, é correto afirmar que:
(A) As imagens formadas por uma lente convergente têm sempre as mesmas características, independentemente da sua posição.
(B) As imagens formadas por uma lente divergente são sempre reais, invertidas e menores que o objeto;
(C) A imagem formada por uma lente sempre é maior que o objeto;
(D) As características da imagem de um objeto produzida por uma lente divergente dependem da posição desse objeto em relação à lente;
(E) As características da imagem formada por uma lente convergente dependem da posição em que o objeto localiza-se em relação a ela.
Resolução:
A imagem formada por uma lente convergente possui suas características definidas pela posição do objeto.Já a imagem de uma lente divergente preserva suas características e é sempre virtual, direita e menor que o objeto.
Alternativa: E 

04) (PUC-SP) Uma lente de vidro cujos bordos são mais espessos que a parte central:
(A) deve ser divergente
(B) deve ser convergente
(C) no ar, é sempre divergente
(D) mergulhada num líquido, torna-se divergenteE) nunca é divergente
Resolução:
As lentes de vidro com bordas espessas são sempre divergentes no ar, pois o índice de refração do vidro é maior que o índice de refração do ar.
Alternativa: C 

05) (UFRS) A figura representa uma lente esférica delgada de distância focal f. Um objeto real é colocado à esquerda da lente, em uma posição tal que sua imagem real se forma à direita dela.
Para que o tamanho dessa imagem seja igual ao tamanho do objeto, este deve ser colocado:
(A) à esquerda de G.
(B) em G.
(C) entre G e F.
(D) em F.
(E) entre F e a lente.
Resolução:
De acordo com a equação do aumento, para que objeto e imagem possuam mesmo tamanho, é necessário que suas distâncias em relação ao centro óptico da lente sejam as mesmas, logo, P = P'. Sendo assim, ao aplicar a equação de Gauss e chamando P e P' de x, teremos:

O objeto deve estar sobre o centro de curvatura da lente, pois esse é o ponto que corresponde ao dobro do foco, representado na figura por G.
Alternativa: B 
06) A figura abaixo mostra um objeto posicionado diante de uma lente esférica convergente e de distância focal igual a 10 cm. Determine a posição e o tipo da imagem formada do objeto.(A) 2 cm, real
(B) 3 cm, real
(C) 5 cm, virtual
(D) 4 cm, virtual
(E) 10 cm, virtual.
Resolução:
Dados:Foco: F = 10 cmPosição do objeto: P = 5 cmAplicando a equação de Gauss, temos:
Como P' é negativo, a imagem é virtual e está a 10 cm da lente.Alternativa: E 

07) (UFRJ) É sabido que lentes descartáveis ou lentes usadas nos óculos tradicionais servem para corrigir dificuldades na formação de imagens no globo ocular e que desviam a trajetória inicial do feixe de luz incidente na direção da retina. Sendo assim, o fenômeno físico que está envolvido quando a luz atravessa as lentes é a:
(A) reflexão especular.
(B) difração luminosa.
(C) dispersão.
(D) difusão.
(E) refração luminosa.
Resolução:
Ao tocar a lente, a luz sofre duas mudanças de meio consecutivas (ar – lente e lente – ar), o que caracteriza o fenômeno da refração luminosa.Alternativa: E 

DALDONISTMO
daltonismo é a incapacidade ou diminuição da capacidade de ver a cor ou perceber as diferenças de cor em condições normais de iluminação.
daltónico é o indivíduo que padece de daltonismo, ou seja, significa que é incapaz ou tem dificuldade em distinguir as diferenças de cor. Por este motivo, a visão de um daltónico é, muitas vezes, apelidada de “cegueira para cores” ou “deficiência de visão das cores”.
Alguém com visão normal pode identificar e distinguir 150 tons de cores diferentes, no entanto, no daltónico este número começa a cair à medida que tem menos possibilidades de criar misturas de cores.

O daltonismo afeta uma percentagem significativa da população. Existe a possibilidade de as pessoas daltónicas pertencerem a ambos os sexos.
Contudo, o daltonismo em homens (masculino) é mais frequente que o daltonismo em mulheres (feminino), devido à hereditariedade e aos cromossomas envolvidos.
Para sabe mais sobre o daltonismo acesse: https://www.saudebemestar.pt/pt/clinica/oftalmologia/daltonismo/


QUESTÕES RESOLVIDAS
01) Um espelho plano vertical conjuga a imagem de um observador parado, situado à 1 m do espelho. Afastando 2 m o espelho do ponto onde se encontrava, que distância passa a separar a primeira da segunda imagem?
Resolução:
Em um espelho plano a distância do objeto ao espelho é igual a distância do espelho a imagem.
x = 2 . 2   →   4 m
Resposta: o observador ficará a 4m de distancia da sua imagem.

02) (Enem/2015) Entre os anos de 1028 e 1038, Alhazen (lbn al-Haytham:965-1040 d.C.) escreveu sua principal obra, o Livro da Óptica, que, com base em experimentos, explicava o funcionamento da visão e outros aspectos da ótica, por exemplo, o funcionamento da câmara escura. O livro foi traduzido e incorporado aos conhecimentos científicos ocidentais pelos europeus. Na figura, retirada dessa obra, é representada a imagem invertida de edificações em tecido utilizado como anteparo.
Se fizermos uma analogia entre a ilustração e o olho humano, o tecido corresponde ao(à)
(A) íris   
(B) retina    
(C) pupila   
(D) córnea    
(E) cristalino
Resolução:
O tecido corresponde à retina: de um objeto real forma-se uma imagem real, invertida e reduzida, no fundo do olho, sobre a retina. A retina é constituída de células nervosas sensíveis à luz, que transmitem ao cérebro as sensações visuais.

Resposta: B

03) (Fuvest) Estudos recentes parecem indicar que o formato do olho humano e a visão são influenciados pela quantidade da substância X, sintetizada pelo organismo. A produção dessa substância é favorecida pela luz solar, e crianças que fazem poucas atividades ao ar livre tendem a desenvolver dificuldade para enxergar objetos distantes. Essa disfunção ocular é comumente chamada de miopia.
Considere a fórmula estrutural da substância X e os diferentes formatos de olho:
Observação: As linhas tracejadas representam o feixe de luz incidente no olho.
Com base nessas informações, conclui-se que a miopia poderá atingir crianças cujo organismo venha a produzir _________ X em quantidade insuficiente, levando à formação de olho do tipo _________
As lacunas da frase acima devem ser preenchidas, respectivamente, por
(A) o aminoácido; III.
(B) a amina; II.
(C) o aminoácido; I.
(D) o fenol; I.
(E) a amina; III.
Alternativa: E

04) (UF – ACRE) Sentado na cadeira da barbearia, um rapaz olha no espelho a imagem do barbeiro, em pé atrás dele. As dimensões relevantes são dadas na figura. A que distância (horizontal) dos olhos do rapaz fica a imagem do barbeiro?
(A) 0,50m
(B) 0,80m
(C) 1,3m
(D) 1,6m
(E) 2,1m
Resolução:
Em um espelho plano a imagem se forma atrás do espelho e a distância do objeto ao espelho e igual a distância do espelho a imagem. Nesse caso, a distância dos olhos do barbeiro até a imagem dos seus olhos e de 1,3 + 1,3 = 2,60. Observando que a rapaz esta 50 cm a frente do barbeiro temos:
2,10 - 0,50 = 2,10 m
Alternativa: E

05) (UNIP) Os dois espelhos planos perpendiculares E e F da figura abaixo conjugam do objeto A três imagens BC e D.
Se os espelhos F se transladam com velocidade de módulo 3,0 cm/s e 4 cm/s, respectivamente, a imagem se movimenta com velocidade de módulo igual a:
(A) 30 cm/s
(B) 20 cm/s
(C) 5,0 cm/s
(D) 7,0 cm/s
(E) 10 cm/s
Resolução:
Nesse caso que a afastamento e a distância da hipotenusa. Aplicando o conceito de hipotenusa temos:
vh 2 = vE2 + vF2
vh 2 = 32 + 42
vh 2 = 9 + 16
vh 2 = 25
vh = 5m/s           
Observação: cada espelho se afasta com velocidade de 5 m/s portanto a velocidade de afastamento da imagem D é de 10 m/s.
Alternativa: E

06) (Enem/2014) As lentes fotocromáticas escurecem quando expostas à luz solar por causa de reações químicas reversíveis entre uma espécie incolor e outra colorida. Diversas reações podem ser utilizadas, e a escolha do melhor reagente para esse fim se baseia em três principais aspectos: (i) o quanto escurece a lente; (ii) o tempo de escurecimento quando exposta à luz solar; e (iii) o tempo de esmaecimento em ambiente sem forte luz solar. A transmitância indica a razão entre a quantidade de luz que atravessa o meio e a quantidade de luz que incide sobre ele. Durante um teste de controle para o desenvolvimento de novas lentes fotocromáticas, foram analisadas cinco amostras que utilizam reagentes químicos diferentes. No quadro, são apresentados os resultados.
Considerando os três aspectos, qual é a melhor amostra de lente fotocromática para se utilizar em óculos?
(A) 1     
(B) 2     
(C) 3     
(D) 4     
(E) 5
Resolução:
A melhor amostra de lente fotocromática deve ter os tempos de escurecimento e de esmaecimento mínimos (20s e 30s, conforme a tabela). A transmitância média (quantidade de luz que atravessa o meio em relação à quantidade de luz que incide sobre ele), deve ser máxima em ambientes escuros e mínima em ambientes claros.
O valor de 50% de transmitância média contempla os dois casos.
Resposta: C

07) (Enem) Alguns povos indígenas ainda preservam suas tradições, realizando a pesca com lanças, demonstrando uma notável habilidade. Para fisgar um peixe em um lago com águas tranquilas o índio deve mirar abaixo da posição em que enxerga o peixe.
Ele deve proceder dessa forma porque os raios de luz
(A) refletidos pelo peixe não descrevem uma trajetória retilínea no interior da água. 
(B) emitidos pelos olhos do índio desviam sua trajetória quando passam do ar para a água. 
(C) espalhados pelo peixe são refletidos pela superfície da água.
(D) emitidos pelos olhos do índio são espalhados pela superfície da água.
(E) refletidos pelo peixe desviam sua trajetória quando passam da água para o ar. 
Resolução:
Ver o peixe, assim como todos os demais objetos, só é possível se ele refletir a luz que incide sobre ele. Portanto, a luz primeiramente é refletida. Em seguida, a luz passa da água para o ar, o que significa que ela muda de meio de propagação. Sempre que isso acontece, há o fenômeno da refração, que é a mudança da direção e da velocidade da luz ao passar de um meio para outro.
Resumindo: a luz que incide no peixe é refletida e, em seguida, muda sua direção ao passar da água para o ar. 
Alternativa: E

08) (Enem) Ao diminuir o tamanho de um orifício atravessado por um feixe de luz, passa menos luz por intervalo de tempo, e próximo da situação de completo fechamento do orifício, verifica-se que a luz apresenta um comportamento como o ilustrado nas figuras. Sabe-se que o som, dentro de suas particularidades, também pode se comportar dessa forma.
Ilustração do comportamento da luz ao passar por orifícios de diferentes larguras
Em qual das situações a seguir está representado o fenômeno descrito no texto? 
(A) Ao se esconder atrás de um muro, um menino ouve a conversa de seus colegas.
(B) Ao gritar diante de um desfiladeiro, uma pessoa ouve a repetição do seu próprio grito. 
(C) Ao encostar o ouvido no chão, um homem percebe o som de uma locomotiva antes de ouvi-lo pelo ar. 
(D) Ao ouvir uma ambulância se aproximando, uma pessoa percebe o som mais agudo do que quando aquela se afasta. 
(E) Ao emitir uma nota musical muito aguda, uma cantora de ópera faz com que uma taça de cristal se despedace.
Resolução:
Cada alternativa da questão diz respeito a algum fenômeno ondulatório. Em geral, essa questão consiste em comparar um fenômeno óptico com fenômenos ondulatórios.
O fenômeno descrito é denominado difração da luz, que consiste na capacidade das ondas de contornar obstáculos com dimensões compatíveis ao seu comprimento de onda. Como o som também é uma onda, ele também sofre difração e pode contornar obstáculos. Esse fato pode ser observado na alternativa A, em que uma pessoa pode ouvir a conversa de outras mesmo estando atrás de obstáculos.
Alternativa: A

09) (Enem) Sabe-se que o olho humano não consegue diferenciar componentes de cores e vê apenas a cor resultante, diferentemente do ouvido, que consegue distinguir, por exemplo, dois instrumentos diferentes tocados simultaneamente. Os raios luminosos do espectro visível, que têm comprimento de onda entre 380 nm e 780 nm, incidem na córnea, passam pelo cristalino e são projetados na retina. Na retina, encontram-se dois tipos de fotorreceptores, os cones e os bastonetes, que convertem a cor e a intensidade da luz recebida em impulsos nervosos. Os cones distinguem as cores primárias: vermelho, verde e azul, e os bastonetes diferenciam apenas níveis de intensidade, sem separar comprimentos de onda. Os impulsos nervosos produzidos são enviados ao cérebro por meio do nervo óptico, para que se dê a percepção da imagem.
Um indivíduo que, por alguma deficiência, não consegue captar as informações transmitidas pelos cones, perceberá um objeto branco, iluminado apenas por luz vermelha, como
(A) um objeto indefinido, pois as células que captam a luz estão inativas.
(B) um objeto rosa, pois haverá mistura da luz vermelha com o branco do objeto.
(C) um objeto verde, pois o olho não consegue diferenciar componentes de cores.
(D) um objeto cinza, pois os bastonetes captam luminosidade, porém não diferenciam cor.
(E) um objeto vermelho, pois a retina capta a luz refletida pelo objeto, transformando-a em vermelho.
Resolução
Quando um indivíduo tem alguma deficiência nos cones — estruturas responsáveis pela diferenciação da cor dos objetos —, somente os bastonetes, que são responsáveis pela percepção da intensidade da cor, são os responsáveis pela visão. Assim, os objetos são vistos apenas na cor cinza.
Alternativa: D

10) (FEI) Uma câmara escura de orifício fornece a imagem de um prédio, o qual se apresenta com altura de 5 cm. Aumentando-se de 100m a distância do prédio à câmara, a imagem reduz-se para 4 cm de altura.
Qual é a distância entre o prédio e a câmara, na primeira posição?
(A) 100 m
(B) 200 m
(C) 300 m
(D) 400 m
(E) 500 m
Resolução
i = 5 cm
O = 4 cm
DI = x + 100
D= 100
Utilizamos a equação:
D= i 
Do  O
+ 100 = 5
     x          4
4 (x + 100) = 5x
4x + 400 = 5x
5x – 4x = 400
x = 400
Alternativa: D

12) Um pesquisador precisava medir a altura de um prédio de vinte andares, porém ele não possuía o instrumento de medida necessário para realizar essa medição. Conhecendo o princípio da propagação retilínea da luz, ele utilizou uma haste de madeira de 1 m de altura e, em seguida, mediu a sombra projetada pela haste, que foi de 20 cm, e a sombra projetada pelo prédio, que foi de 12 m.
Calcule a altura do prédio de acordo com esses dados encontrados pelo pesquisador.
Resolução
o = ?
i = 12 m
p = 1 m
p' = 20 cm = 0,2 m
Utilizamos a expressão:
_i_ = p'
 o      p

Substituindo os dados, temos:
12 0,2
 o      1
o = 12
     0,2
o = 60 m
Resposta: a altura do prédio é 60 m.

13) Um objeto de 8,0 m de altura é colocado na frente de uma câmara escura de orifício a uma distância de 3,0 m. Sabendo que a câmara possui 25 cm de profundidade, calcule o tamanho da imagem formada.
Resolução
o = 8,0 m
p = 3,0 m
p' = 25 cm = 0,25 m
i = ?
Utilizamos a expressão:
_i_ = p'
 o     p

Substituindo os dados, temos:
_i0,25
 8        3
3 i = 8 . 0,25
i = 2
     3
i = 0,67 m
Resposta: i = 0,67 m

14) (Unitau) Dois raios de luz, que se propagam em um meio homogêneo e transparente, interceptam-se em certo ponto. A partir desse ponto, pode-se afirmar que:
(A) os raios luminosos cancelam-se.
(B) mudam a direção de propagação.
(C) continuam propagando-se na mesma direção e sentindo que antes.
(D) propagam-se em trajetórias curvas.
(E) retornam em sentidos opostos.
Resolução
Quando dois raios de luz propagam-se em um meio homogêneo e transparente, eles continuam propagando-se na mesma direção e sentido. Isso ocorre graças ao princípio da independência dos raios luminosos, que afirma que os raios de luz são independentes ao interceptarem-se, pois cada um deles mantém sua trajetória como se os demais não existissem.
Alternativa: C

16) O professor Diminoi pede aos grupos de estudo que apresentem à classe suas principais conclusões sobre os fundamentos para o desenvolvimento do estudo da óptica geométrica.
Grupo I. Os feixes de luz podem apresentar-se em raios paralelos, convergentes ou divergentes.
Grupo II. Os fenômenos de reflexão, refração e absorção ocorrem isoladamente e nunca simultaneamente.
Grupo III. Enquanto num corpo pintado de preto fosco predomina a absorção, em um corpo pintado de branco predomina a difusão.
Grupo IV. Os raios luminosos se propagam em linha reta nos meios homogêneos e transparentes.
São corretas as conclusões dos grupos:
(A) apenas I e III
(B) apenas II e IV
(C) apenas I, III e IV
(D) II, III, IV
(E) I, II, III e IV
Resolução
I – correta
II – Errada. Os fenômenos de reflexão, refração e absorção ocorrem simultaneamente.
III – Correta
IV – Correta
Alternativa:  C

17) Uma pessoa está parada em frente a um grande espelho plano, observando a sua própria imagem, e começa a se lembrar dos conceitos aprendidos no ensino médio, na disciplina de física.
Levando em conta que se trata de um espelho plano, analise as afirmações a seguir:
I. A imagem tem as mesmas dimensões do objeto.
II. A imagem e o objeto estão simetricamente colocados em relação ao plano do espelho.
III. A imagem formada é real e menor que o objeto.
IV. A imagem e o objeto apresentam formas contrárias, isto é, são figuras enantiomorfas.
Estão corretas:
(A) apenas I e II
(B) apenas III e IV
(C) apenas I, II e IV
(D) I, II, III
(E) I, II, III e IV
Resolução
I – correta
II – Correta
III – Errada, porque a imagem no espelho plano é virtual e do mesmo tamanho que o objeto.
IV – Correta
Alternativa: C

18) Em um holofote, a lâmpada é colocada no foco do espelho côncavo para que:
(A) o feixe transmitido pelo espelho seja constituído de raios paralelos.
(B) os raios de luz refletidos pelo espelho convirjam para o alvo.
(C) o feixe transmitido pelo espelho seja mais intenso.
(D) o feixe refletido forme uma imagem virtual.
(E) o feixe refletido seja constituído de raios paralelos.
Resolução
Sabemos que todo raio de luz que passa pelo foco de um espelho côncavo é refletido paralelamente ao eixo principal deste espelho.
Alternativa: E

19) O ângulo entre um raio de luz que incide em um espelho plano e a normal à superfície do espelho é igual a 35°. Para esse caso, o ângulo entre o espelho e o raio refletido é igual a:
(A) 20°
(B) 35°
(C) 45°
(D) 55°
(E) 65°
Resolução
Sabemos, pelas leis da reflexão, que o ângulo de incidência é igual ao ângulo de reflexão:
i = r
logo,
r = 35°
Para encontrar o ângulo entre o espelho e o raio refletido (a), basta encontrar o complemento de r. Pois o ângulo entre a reta normal e o espelho é igual a 90°.
a + r = 90
a + 35 = 90
a = 90 – 35
a = 55°
Alternativa: D

20) A receita de óculos para um míope indica que ele deve usar “lentes de 2,0 graus”, isto é, o valor da vergência das lentes deve ser 2,0 dioptrias. Podemos concluir que as lentes desses óculos devem ser:
(A) convergentes, com 2,0 m de distância focal.
(B) convergentes, com 5,0 cm de distância focal.
(C) divergentes, com 2,0 m de distância focal.
(D) divergentes, com 20 cm de distância focal.
(E) divergentes, com 50 cm de distância focal.
Resolução
Para a correção da miopia utilizamos lentes divergentes. As lentes divergentes vão aproximar da retina as imagens dos objetos distantes. C = -2,0 di
A vergencia (C) de uma lente é dada pelo inverso da distância focal, logo:
C = 1 / f
f = 1 / C
f = 1 / -2
f = - 0,5m ou f = - 50 cm
Alternativa: E

21) A figura ilustra o esquema de um olho de uma pessoa que está tentando observar um objeto que se encontra muito distante. Os raios indicados na figura são provenientes de um ponto desse objeto. Podemos afirmar que se trata de um defeito de visão de um olho.
(A) míope e para corrigi-lo necessita-se de lente convergente.
(B) míope e para corrigi-lo necessita-se de lente divergente.
(C) hipermetrope e para corrigi-lo necessita-se de lente convergente.
(D) hipermetrope e para corrigi-lo necessita-se de lente divergente.
(E) hipermetrope e para corrigi-lo necessita-se de lente bifocal.
Resolução
A miopia é uma deficiência da visão onde, o indivíduo míope não enxerga bem os objetos ao longe. A miopia surge em resultado de um alongamento do globo ocular, do que resulta que a imagem projetada pelo cristalino se forma à frente (antes) da retina como vemos no desenho acima. A correção da miopia é feita comumente com a utilização de lentes divergentes.
Alternativa: B

22) (Fuvest) Admita que o sol subitamente “morresse”, ou seja, sua luz deixasse de ser emitida. Vinte e quatro horas após esse evento, um eventual sobrevivente, olhando para o céu, sem nuvens, veria:
(A) a Lua e estrelas;
(B) somente a Lua;
(C) somente estrelas;
(D) uma completa escuridão;
(E) somente os planetas do sistema solar;
Resolução:
O Sol é uma estrela e as estrelas têm luz própria, são fontes de luz primárias. A Lua é um satélite natural da Terra e não têm luz própria, apenas reflete a luz do Sol, é uma fonte de luz secundária. Os planetas também não têm luz própria, apenas refletem a luz do Sol.
Logo, se o Sol deixasse de emitir luz apenas veríamos as estrelas que são fontes de luz primárias.
Alternativa: C

QUESTÕES DO ENEM/VESTIBULAR
01) (Enem - MEC)

Seu olhar

Na eternidade
Eu quisera ter
Tantos anos-luz
Quantos fosse precisar
Pra cruzar o túnel
Do tempo do seu olhar
(Gilberto Gil, 1984)

Gilberto Gil usa na letra da música a palavra composta anos-luz. O sentido prático em geral não é obrigatoriamente o mesmo que na ciência. Na Física, um ano-luz é uma medida que relaciona a velocidade da luz e o tempo de um ano e que, portanto, se refere a:
(A) tempo
(B) aceleração
(C) distância
(D) velocidade
(E) luminosidade
Resolução
Na letra da música, ano-luz corresponde ao tempo, mas, na verdade, deveria se referir a uma distância.
Alternativa C

02) (UFF) A luz proveniente do Sol demora, aproximadamente, 8 minutos para chegar à Terra. Qual a distância entre esses dois astros celestes?
DADO:
Velocidade da luz = 3 . 105 km/s
Resolução:
1 segundo a luz percorre 3 .105 km
1 minuto a luz percorre 60 . 3 .105 km = 180 ..105 km
8 minutos a luz percorre 8 . 180 .105 km = 1440 . 105 km = 1,44 ..108 km

03) (Enem 2019) Os olhos humanos normalmente têm três tipos de cones responsáveis pela percepção das cores: um tipo para tons vermelhos, um para tons azuis e outro para tons verdes. As diversas cores que enxergamos são o resultado da percepção das cores básicas, como indica a figura.

A protanopia é um tipo de daltonismo em que há diminuição ou ausência de receptores da cor vermelha. Considere um teste com dois voluntários: uma pessoa com visão normal e outra com caso severo de protanopia. Nesse teste, eles devem escrever a cor dos cartões que lhes são mostrados. São utilizadas as cores indicadas na figura.
Para qual cartão os dois voluntários identificarão a mesma cor?
(A)    Vermelho.
(B)    Magenta.
(C)    Amarelo.
(D)    Branco.
(E)    Azul
Resolução:
Devemos mostrar um cartão que não contenha o vermelho em sua composição. Qualquer cartão que só reflita o verde ou o azul, ou que reflita o verde e o azul (ciano) serão identificados pelos dois voluntários com a mesma cor.
Resposta: E

04) (Enem 2019) Quando se considera a extrema velocidade com que a luz se espalha por todos os lados e que, quando vêm de diferentes lugares, mesmo totalmente opostos, [os raios luminosos] se atravessam uns aos outros sem se atrapalharem, compreende-se que, quando vemos um objeto luminoso, isso não poderia ocorrer pelo transporte de uma matéria que venha do objeto até nós, como uma flecha ou bala atravessa o ar; pois certamente isso repugna bastante a essas duas propriedades da luz, principalmente a última.
O texto contesta que concepção acerca do comportamento da luz?
(A)    O entendimento de que a luz precisa de um meio de propagação, difundido pelos defensores da existência do éter.
(B)    O modelo ondulatório para a luz, o qual considera a possibilidade de interferência entre feixes luminosos.
(C)    O modelo corpuscular defendido por Newton, que descreve a luz como um feixe de partículas.
(D)    A crença na velocidade infinita da luz, defendida pela maioria dos filósofos gregos.
(E)    A ideia defendida pelos gregos de que a luz era produzida pelos olhos.
Resolução:
Newton conjecturava sobre a luz como uma formação de partículas, ao contrário do Princípio de Huygens, que considera a luz como uma onda.
Resposta: C

05) (Enem 2016) Algumas crianças, ao brincarem de esconde-esconde, tapam os olhos com as mãos, acreditando que, ao adotarem tal procedimento, não poderão ser vistas. Essa percepção da criança contraria o conhecimento científico porque, para serem vistos, os objetos
(A)    refletem partículas de luz (fótons), que atingem os olhos.
(B)    geram partículas de luz (fótons), convertidas pela fonte externa.
(C)    são atingidos por partículas de luz (fótons), emitidas pelos olhos.
(D)    refletem partículas de luz (fótons), que se chocam com os fótons emitidos pelos olhos.
(E)    são atingidos pelas partículas de luz (fótons), emitidas pela fonte externa e pelos olhos.
Resolução:
Nosso corpo, sendo uma fonte de luz primária, reflete a luz emitida por outras fontes luminosas, e só seremos vistos por alguém se as luzes refletidas pelos nossos corpos atingirem os olhos de um observador.
Resposta: A

06) (Enem 2019) Um estudante leu em um site da internet que os povos antigos determinavam a duração das estações do ano observando a variação do tamanho da sombra de uma haste vertical projetada no solo. Isso ocorria porque, se registrarmos o tamanho da menor sombra ao longo de um dia (ao meio-dia solar), esse valor varia ao longo do ano, o que permitiu aos antigos usar esse instrumento rudimentar como um calendário solar primitivo. O estudante também leu que, ao longo de um ano (sempre ao meio-dia solar): (I) a sombra é máxima no solstício de inverno; e (II) a sombra é mínima no solstício de verão.
O estudante, que morava em Macapá (na Linha do Equador), ficou intrigado com essas afirmações e resolveu verificar se elas eram verdadeiras em diferentes regiões do mundo. Contatou seus amigos virtuais em Salvador (região tropical) e Porto Alegre (região temperada) e pediu que eles registrassem o tamanho da menor sombra de uma haste vertical padronizada, ao longo do dia, durante um ano. Os resultados encontrados estão mostrados esquematicamente no gráfico (SV: Solstício de Verão; SI: Solstício de Inverno; E: Equinócio):

Qual(is) cidade(s) indicada(s) no texto e no gráfico contradiz(em) a afirmação II?
(A)    Salvador.
(B)    Porto Alegre.
(C)    Macapá e Salvador.
(D)    Macapá e Porto Alegre.
(E)    Porto Alegre e Salvador.
Resolução:
Verificando o gráfico, percebemos que em Salvador a menor sombra foi formada no meio da primavera e no meio do verão. Já em Macapá, a menor sombra foi formada no início da primavera e no início do outono. Ambas as situações estão em desacordo com a afirmação II.
Resposta: C

07) (Enem 2014) Folhas de papel, como as utilizadas para a impressão de documentos, são opacas e permeáveis aos líquidos. Esse material é constituído de microfibras entrelaçadas de celulose, que são transparentes à luz. Quando sobre elas se derrama glicerina, elas se tornam translúcidas. Uma imagem da superfície de uma folha de papel, ampliada por um microscópio eletrônico de varredura, pode ser vista na figura. No quadro é apresentada a razão (n) entre a velocidade da luz no vácuo e no respectivo material (celulose, glicerina ou ar).
Nessa situação, o papel se tornou translúcido porque a luz é
(A)    mais refletida.
(B)    mais absorvida.
(C)    mais espalhada.
(D)    menos refratada.
(E)    menos transmitida.
Resolução:
Nos meios translúcidos, raios de luz que incidem em sua superfície paralelos entre si perdem esse paralelismo ao entrar no material. Além disso, quanto mais esses raios se espalham, mais translúcido é o meio material.
Resposta: D

08) (Enem 2019) A figura mostra, de forma esquemática, uma representação comum em diversos livros e textos sobre eclipses. Apenas analisando essa figura, um estudante pode concluir que os eclipses podem ocorrer duas vezes a cada volta completa da lua em torno da Terra. Apesar de a figura levar a essa percepção, algumas informações adicionais são necessárias para se concluir que nem o eclipse solar, nem o lunar ocorrem com tal periodicidade.
A periodicidade dos eclipses ser diferente da possível percepção do estudante ocorre em razão de
(A)    eclipses noturnos serem imperceptíveis da Terra.
(B)    planos das órbitas da Terra e da lua serem diferentes.
(C)    distância entre a Terra e a lua variar ao longo da órbita.
(D)    eclipses serem visíveis apenas em parte da superfície da Terra.
(E)    o Sol ser uma fonte de luz extensa comparado ao tamanho da lua.
Resolução:
Terra faz sua órbita ao redor do Sol em um plano geométrico diferente daquele em que a lua faz sua órbita ao redor da Terra.
Resposta: B

09) (Fatec 2020) Em 2019 comemora-se, em todo o mundo, o centenário das observações astronômicas realizadas na cidade de Sobral (CE), durante o eclipse solar de 29 de maio de 1919. As medidas da deflexão da luz decorrentes das estrelas na borda do Sol constituíram uma prova fundamental para a confirmação da Teoria da Relatividade Geral do físico Albert Einstein.
Sobre o eclipse referido, é correto afirmar que
(A)    as regiões de eclipse solar total comprovam a Terra ser plana e as de eclipse solar parcial a lua ser esférica.
(B)    as regiões de eclipse solar total equivalem às penumbras e as de eclipse solar parcial, às sombras.
(C)    a Terra se coloca entre o Sol e a lua, projetando, assim, a sombra da Terra na lua.
(D)    o Sol se coloca entre a Terra e a lua, projetando, assim, a sombra do Sol na Terra.
(E)    a lua se coloca entre o Sol e a Terra, projetando, assim, a sombra da lua na Terra.
Resolução:
Em um eclipse solar, a lua fica entre o Sol e a Terra alinhados.
Resposta: E

10) (PUC-SP) A ocorrência do eclipse da figura só foi possível porque a lua, além de estar alinhada com o Sol e a Terra, estava na fase
Chester, Illinois, Estados Unidos. O ECLIPSE SOLAR TOTAL É VISTO DA PONTE DO RIO MARY
http://www.msn.com/pt-br/clima/noticias-do-clima/imagens
Consultado em: 31/08/2017
(A)    quarto crescente.
(B)    quarto minguante.
(C)    nova.
(D)    cheia.
Resolução:
O eclipse solar só ocorre na lua nova.
Resposta: C

11) (Enem) 
A ilustração representa uma das mais conhecidas obras do artista gráfico holandês M. C. Escher. Seu trabalho tem como características as figuras geométricas e ilusões de óptica.
Pelas características da imagem formada na gravura, o artista representou um espelho esférico do tipo
(A)    convexo, pois as imagens de todos os objetos, formadas na esfera, inclusive a do artista, são virtuais.
(B)    côncavo, pois as imagens são direitas, indicando que todos os objetos visualizados estão entre o foco e o espelho.
(C)    côncavo, devido ao pequeno campo de visão, não é possível observar todos os detalhes do local onde se encontra o artista.
(D)   convexo, pois as imagens são formadas pelo cruzamento dos raios de luz refletidos pela esfera, por isso as imagens são direitas e não invertidas.
(E)    côncavo, devido às imagens formadas por este espelho serem todas reais, ou seja, formadas pelo cruzamento dos raios de luz refletidos pela esfera.
Resolução:
No espelho convexo, a imagem sempre será virtual, menor que o objeto e direita.
Resposta: A

12) (Enem) A figura mostra uma superfície refletora de formato parabólico que tem sido utilizada como um fogão solar. Esse dispositivo é montado de tal forma que a superfície fique posicionada sempre voltada para o Sol. Neste, a panela deve ser colocada em um ponto determinado para maior eficiência do fogão.
Considerando que a panela esteja posicionada no ponto citado, a maior eficiência ocorre porque os raios solares
(A)    refletidos passam por esse ponto, definido como ponto de reflexão.
(B)    incidentes passam por esse ponto, definido como vértice da parábola.
(C)    refletidos se concentram nesse ponto, definido como foco da parábola.
(D)    incidentes se concentram nesse ponto, definido como ponto de incidência.
(E)    incidentes e refletidos se interceptam nesse ponto, definido como centro de curvatura.
Resolução:
Todos os raios de luz que incidem em um espelho côncavo paralelamente ao seu eixo principal, após a reflexão, passam pelo foco principal desse espelho.
Resposta: C

16) (Encceja) No deserto do Saara, ocorrem fenômenos em que paisagens são modificadas pela ação direta da luz solar. Esse fenômeno é conhecido como miragem. Embora algumas pessoas acreditem ser alucinações, nada mais são do que um fenômeno físico real. O aquecimento sofrido pela areia faz com que o ar próximo se aqueça, diminuindo sua densidade, fazendo com que a luz se desvie, dando-nos a impressão de uma nova paisagem.
Esse fenômeno físico é descrito como
(A)    interferência.
(B)    refração.
(C)    reflexão.
(D)    difração.
Resolução:
refração da luz pode nos dar a falsa impressão de que os raios de luz estão sendo refletidos por uma poça de água.
Resposta: B

15) (Enem) No hemisfério Sul, o solstício de verão (momento em que os raios solares incidem verticalmente sobre quem se encontra sobre o Trópico de Capricórnio) ocorre no dia 21 ou 23 de dezembro. Nessa data, o dia tem o maior período de presença de luz solar. A figura mostra a trajetória da luz solar nas proximidades do planeta Terra quando ocorre o fenômeno óptico que possibilita que o Sol seja visto por mais tempo pelo observador.
Qual é o fenômeno óptico mostrado na figura?
(A)    A refração da luz solar ao atravessar camadas de ar com diferentes densidades.
(B)    A polarização da luz solar ao incidir sobre a superfície dos oceanos.
(C)    A reflexão da luz solar nas camadas mais altas da ionosfera.
(D)    A difração da luz solar ao contornar a superfície da Terra.
(E)    O espalhamento da luz solar ao atravessar a atmosfera.
Resolução:
Ao passar de um meio menos refringente para um meio mais refringente, a luz desvia sua trajetória, se aproximando da reta normal à superfície de separação desses meios.
Resposta: A

16) (Enem) A fotografia feita sob luz polarizada é usada por dermatologistas para diagnósticos. Isso permite ver detalhes da superfície da pele que não são visíveis com o reflexo da luz branca comum. Para se obter luz polarizada, pode-se utilizar a luz transmitida por um polaroide ou a luz refletida por uma superfície na condição de Brewster, como mostra a figura. Nessa situação, o feixe da luz refratada forma um ângulo de 90° com o feixe da luz refletida, fenômeno conhecido como Lei de Brewster. Nesse caso, o ângulo de incidência   , também chamado de ângulo de polarização, e o ângulo de refração    estão em conformidade com a Lei de Snell.
Considere um feixe de luz não polarizada proveniente de um meio com índice de refração igual a 1, que incide sobre uma lâmina e faz um ângulo de refração  de 30°.
Nessa situação, qual deve ser o índice de refração da lâmina para que o feixe de luz seja polarizado?
(A) √3
(B) √3/3
(C) 2
(D) 1/2
(E) √3/2
Resolução:

Resposta: A

17) (Enem) Uma proposta de dispositivo capaz de indicar a qualidade da gasolina vendida em postos e, consequentemente, evitar fraudes, poderia utilizar o conceito de refração luminosa. Nesse sentido, a gasolina não adulterada, na temperatura ambiente, apresenta razão entre os senos dos raios incidente e refratado igual a 1,4. Desse modo, fazendo incidir o feixe de luz proveniente do ar com um ângulo fixo e maior que zero, qualquer modificação no ângulo do feixe refratado indicará adulteração no combustível.
Em uma fiscalização rotineira, o teste apresentou o valor de 1,9. Qual foi o comportamento do raio refratado?
(A)    Mudou de sentido.
(B)    Sofreu reflexão total.
(C)    Atingiu o valor do ângulo limite.
(D)    Direcionou-se para a superfície de separação.
(E)    Aproximou-se da normal à superfície de separação.
Resolução:
Ao passar de um meio menos refringente para um meio mais refringente, a luz desvia sua trajetória, se aproximando da reta normal à superfície de separação desses meios.
Resposta: E

18) (Enem) A maioria das pessoas fica com a visão embaçada ao abrir os olhos debaixo dʼágua. Mas há uma exceção: o povo moken, que habita a costa da Tailândia. Essa característica se deve principalmente à adaptabilidade do olho e à plasticidade do cérebro, o que significa que você também, com algum treinamento, poderia enxergar relativamente bem debaixo dʼágua. Estudos mostraram que as pupilas de olhos de indivíduos moken sofrem redução significativa debaixo dʼágua, o que faz com que os raios luminosos incidam quase paralelamente ao eixo óptico da pupila.
A acuidade visual associada à redução das pupilas é fisicamente explicada pela diminuição
(A)    da intensidade luminosa incidente na retina.
(B)    da difração dos feixes luminosos que atravessam a pupila.
(C)    da intensidade dos feixes luminosos em uma direção por polarização.
(D)    do desvio dos feixes luminosos refratados no interior do olho.
(E)    das reflexões dos feixes luminosos no interior do olho.
Resolução:
Os raios de luz que incidem junto à reta normal à superfície de separação dos meios não desviam sua trajetória ao sofrerem refração.
Resposta: D

19) (Enem) A retina é um tecido sensível à luz, localizado na parte posterior do olho, onde ocorre o processo de formação de imagem. Nesse tecido, encontram-se vários tipos celulares específicos. Um desses tipos celulares são os cones, os quais convertem os diferentes comprimentos de onda da luz visível em sinais elétricos, que são transmitidos pelo nervo óptico até o cérebro.
Em relação à visão, a degeneração desse tipo celular irá
(A)    comprometer a capacidade de visão em cores.
(B)    impedir a projeção dos raios luminosos na retina.
(C)    provocar a formação de imagens invertidas na retina.
(D)    causar dificuldade de visualização de objetos próximos.
(E)    acarretar a perda da capacidade de alterar o diâmetro da pupila.
Resolução:
As células chamadas cone nos conferem sensibilidade às cores.
Resposta: A

20) (Enem) Entre os anos de 1028 e 1038, Alhazen (Ibn al-Haytham; 965–1040 d.C.) escreveu sua principal obra, o Livro da Óptica, que, com base em experimentos, explicava o funcionamento da visão e outros aspectos da óptica, por exemplo, o funcionamento da câmara escura. O livro foi traduzido e incorporado aos conhecimentos científicos ocidentais pelos europeus. Na figura, retirada dessa obra, é representada a imagem invertida de edificações em um tecido utilizado como anteparo.
Se fizermos uma analogia entre a ilustração e o olho humano, o tecido corresponde ao(à)
(A)    íris.
(B)    retina.
(C)    pupila.
(D)    córnea.
(E)    cristalino.
Resolução:
O tecido da figura está fazendo o papel de uma tela com a imagem projetada, e dentro dos nossos olhos a imagem é projetada na retina.
Resposta: B

21) (Enem) O avanço tecnológico da medicina propicia o desenvolvimento de tratamento para diversas doenças, como as relacionadas à visão. As correções que utilizam laser para o tratamento da miopia são consideradas seguras até 12 dioptrias, dependendo da espessura e curvatura da córnea. Para valores de dioptria superiores a esse, o implante de lentes intraoculares é mais indicado. Essas lentes, conhecidas como lentes fácicas (LF), são implantadas junto à córnea, antecedendo o cristalino (C), sem que esse precise ser removido, formando a imagem correta sobre a retina (R).
O comportamento de um feixe de luz incidindo no olho que possui um implante de lentes fácicas para correção do problema de visão apresentado é esquematizado por

Resolução:
A lente LF deve abrir o feixe de luz (lente divergente), para corrigir a miopia. Já a lente C (cristalino) deve fechar o feixe de luz (lente convergente), para formar a imagem na retina R.
Resposta: B

22) (Enem 2011) Indivíduos míopes têm dificuldade de enxergar objetos distantes. Para correção desse problema com lentes, o oftalmologista deve medir a distância máxima que o indivíduo pode enxergar nitidamente, que corresponde à distância focal da lente. A vergência (V) de uma lente é numericamente igual ao inverso da distância focal (f), dada em metros (V = 1/f). A vergência é medida em dioptria (di), comumente denominada de graus de uma lente.
Se a distância máxima a que o indivíduo míope enxerga nitidamente for 50 cm, para corrigir o problema, o oftalmologista receitará lentes de vergência
(A) – 2,00 di.
(B) – 0,02 di.
(C) 0,02 di.
(D) 0,20 di.
(E) 2,00 di.
Resolução:

Devemos dividir a distância focal por 100 para convertê-la em metros e colocar um sinal negativo, pois, no referencial de Gauss, lentes divergentes possuem uma distância focal negativa.
 
Resposta: A
 

Continua...